Что сначала делать сложение или умножение: Порядок выполнения действий в выражениях без скобок и со скобками

Содержание

Что первое решается умножение или сложение

Вот вам очень про­стой мате­ма­ти­че­ский при­мер:

8 / 2(2 + 2)

Вы уди­ви­тесь, но боль­шин­ство людей не смо­гут пра­виль­но это посчи­тать. Посчи­тай­те сами и потом смот­ри­те пра­виль­ный ответ:

В интер­не­те мно­го спо­ров про такие при­ме­ры, поэто­му мы реши­ли разо­брать­ся, какие ошиб­ки совер­ша­ют чаще все­го и поче­му мно­гие счи­та­ют непра­виль­но. Для реше­ния нам пона­до­бят­ся три мате­ма­ти­че­ских пра­ви­ла:

  1. То, что в скоб­ках, выпол­ня­ет­ся в первую оче­редь. Если ско­бок несколь­ко, они выпол­ня­ют­ся сле­ва напра­во.
  2. При отсут­ствии ско­бок мате­ма­ти­че­ские дей­ствия выпол­ня­ют­ся сле­ва напра­во, сна­ча­ла умно­же­ние и деле­ние, потом — сло­же­ние и вычи­та­ние.
  3. Меж­ду мно­жи­те­лем и скоб­кой (или дву­мя скоб­ка­ми) может опус­кать­ся знак умно­же­ния.

Раз­бе­рём подроб­нее, что это зна­чит в нашем слу­чае.

1. То, что в скоб­ках, выпол­ня­ет­ся в первую оче­редь.

То есть в нашем при­ме­ре, вне зави­си­мо­сти от чего угод­но, сна­ча­ла схлоп­нут­ся скоб­ки:

8 / 2(2 + 2) → 8 / 2(4)

2. Меж­ду чис­лом и скоб­кой мож­но опу­стить знак умно­же­ния. У нас перед скоб­кой двой­ка, то есть мож­но сде­лать такую заме­ну:

3. Мате­ма­ти­че­ские дей­ствия при отсут­ствии ско­бок выпол­ня­ют­ся сле­ва напра­во: как при чте­нии, сна­ча­ла умно­же­ние и деле­ние, потом — сло­же­ние и вычи­та­ние. Умно­же­ние и деле­ние име­ют оди­на­ко­вый при­о­ри­тет. Нет тако­го, что сна­ча­ла все­гда дела­ет­ся умно­же­ние, затем деле­ние, или наобо­рот. Со сло­же­ни­ем и вычи­та­ни­ем то же самое.

Неко­то­рые счи­та­ют, что раз мно­жи­те­ли были напи­са­ны близ­ко друг к дру­гу (когда там сто­я­ли скоб­ки), то оно выпол­ня­ет­ся в первую оче­редь, ссы­ла­ясь при этом на раз­ные мето­ди­че­ские посо­бия. На самом деле это не так, и нет тако­го скры­то­го умно­же­ния, кото­рое име­ет при­о­ри­тет над дру­гим умно­же­ни­ем или деле­ни­ем. Это такое же умно­же­ние, как и осталь­ные, и оно дела­ет­ся в общем поряд­ке — как и при­ня­то во всём мате­ма­ти­че­ском мире.

Полу­ча­ет­ся, что нам сна­ча­ла надо сло­жить 2 + 2 в скоб­ках, потом 8 раз­де­лить на 2, и полу­чен­ный резуль­тат умно­жить на то, что в скоб­ках:

8 / 2 × (2 + 2) = 8 / 2 × 4 = 4 × 4 = 16

Кста­ти, если на айфоне запи­сать это выра­же­ние точ­но так же, как в усло­вии, теле­фон тоже даст пра­виль­ный ответ.

А инже­нер­ный каль­ку­ля­тор на Windows 10 так запи­сы­вать не уме­ет и про­пус­ка­ет первую двойку-множитель. Попро­буй­те сами 🙂

Тут в тред вры­ва­ют­ся мате­ма­ти­ки и с воп­ля­ми «Шустеф!» пояс­ня­ют кри­ком:

«В АЛГЕБРЕ ТОТ ЖЕ ПОРЯДОК ДЕЙСТВИЙ, ЧТО И В АРИФМЕТИКЕ, но есть исклю­че­ние: в алгеб­ре знак умно­же­ния свя­зы­ва­ет ком­по­нен­ты дей­ствия силь­нее, чем знак деле­ния, поэто­му знак умно­же­ния опус­ка­ет­ся. Напри­мер, a:b·c= a: (b·c)».

Этот текст из «Мето­ди­ки пре­по­да­ва­ния алгеб­ры», курс лек­ций, Шустеф М. Ф., 1967 год. (стр. 43)

Раз в спор­ном при­ме­ре знак умно­же­ния опу­щен, то спор­ный при­мер алгеб­ра­и­че­ский, а зна­чит, сна­ча­ла умно­жа­ем 2 на 4, а потом 8 делим на 8!

А вот как на это отве­ча­ют те, кто дей­стви­тель­но в теме и не ленит­ся пол­но­стью посмот­реть пер­во­ис­точ­ник:

«Для устра­не­ния недо­ра­зу­ме­ний В. Л. Гон­ча­ров ука­зы­ва­ет, что пред­по­чти­тель­нее поль­зо­вать­ся в каче­стве зна­ка деле­ния чер­той и ста­вить скоб­ки [87]. П. С. Алек­сан­дров и А. Н. Кол­мо­го­ров [59] пред­ло­жи­ли изме­нить поря­док дей­ствий в ариф­ме­ти­ке и решать, напри­мер, так: 80:20×2=80:40=2 вме­сто обыч­но­го: 80:20×2=4×2=8. Одна­ко это пред­ло­же­ние не нашло под­держ­ки».

Если апел­ли­ро­вать к Фри­де Мак­совне Шустеф, то выхо­дит, что:

  1. В. Л. Гон­ча­ров гово­рит так: «Ребя­та, исполь­зуй­те чер­ту и ставь­те скоб­ки, что­бы ни у кого не было вопро­сов про при­о­ри­тет».
  2. Если у нас всё же бит­ва ариф­ме­ти­ки и алгеб­ры, то, по П. С. Алек­сан­дро­ву и А. Н. Кол­мо­го­ро­ву, при­мер нуж­но решать сле­ва напра­во, как обыч­но. Они, конеч­но, пред­ло­жи­ли решать такое по-другому, но науч­ное сооб­ще­ство их не под­дер­жа­ло.

Самое инте­рес­ное, что даль­ше в при­ме­рах Фри­да Мак­сов­на поль­зу­ет­ся как раз пра­виль­ным поряд­ком дей­ствий, объ­яс­няя реше­ние. Даже там, где есть умно­же­ние на скоб­ку с опу­щен­ным зна­ком, она выпол­ня­ет дей­ствия сле­ва напра­во.

С самого начала следует напомнить, чтобы потом не путаться: есть цифры – их 10. От 0 до 9. Есть числа, и они состоят их цифр. Чисел бесконечно много. Точно больше, чем звезд на небе.

Математическое выражение − это записанное с помощью математических символов наставление, какие действия нужно произвести с числами, чтобы получить результат. Не «выйти» на искомый результат, как в статистике, а узнать, сколько их точно было. А вот чего и когда было − уже не входит в сферу интересов арифметики. При этом важно не ошибиться в последовательности действий, что сначала – сложение или умножение? Выражение в школе иногда называют «пример».

Вам будет интересно: Консилиум – это не приговор

Сложение и вычитание

Какие же действия можно произвести с числами? Есть два базовых. Это сложение и вычитание. Все остальные действия построены на этих двух.

Самое простое человеческое действие: взять две кучки камней и смешать их в одну. Это и есть сложение. Для того чтобы получить результат такого действия, можно даже не знать, что такое сложение. Достаточно просто взять кучку камней у Пети и кучку камней у Васи. Сложить все вместе, посчитать все заново. Новый результат последовательного счета камней из новой кучки − это и есть сумма.

Вам будет интересно: Остеоны или система Гаверсова

Точно так же можно не знать, что такое вычитание, просто взять и разделить кучу камней на две части или забрать из кучи какое-то количество камней. Вот и останется в куче то, что называется разностью. Забрать можно только то, что есть в куче. Кредит и прочие экономические термины в данной статье не рассматриваются.

Чтобы не пересчитывать каждый раз камни, ведь бывает, что их много и они тяжелые, придумали математические действия: сложение и вычитание. И для этих действий придумали технику вычислений.

Сумма двух любых цифр тупо заучиваются без всякой техники. 2 плюс 5 равно семь. Посчитать можно на счетных палочках, камнях, рыбьих головах – результат одинаковый. Положить сначала 2 палочки, потом 5, а потом посчитать все вместе. Другого способа нет.

Те, кто поумнее, обычно это кассиры и студенты, заучивают больше, не только сумму двух цифр, но и суммы чисел. Но самое главное, они могут складывать числа в уме, используя разные методики. Это называется навыком устного счета.

Вам будет интересно: Тореро – это. Значение слова

Для сложения чисел, состоящих из десятков, сотен, тысяч и еще больших разрядов, используют специальные техники − сложение столбиком или калькулятор. С калькулятором можно не уметь складывать даже цифры, да и читать дальше не нужно.

Сложение столбиком −­­­­­­ это метод, который позволяет складывать большие (многоразрядные) числа, выучив только результаты сложения цифр. При сложении столбиком последовательно складываются соответствующие десятичные разряды двух чисел (то есть фактически две цифры), если результат сложения двух цифр превышает 10, то учитывается только последний разряд этой суммы – единицы числа, а к сумме следующих разрядов добавляется 1.

Умножение

Математики любят группировать похожие действия для упрощения расчетов. Так и операция умножения является группировкой одинаковых действий – сложения одинаковых чисел. Любое произведение N x M − есть N операций сложения чисел M. Это всего лишь форма записи сложения одинаковых слагаемых.

Для вычисления произведения используется такой же метод – сначала тупо заучивается таблица умножения цифр друг на друга, а потом применяется метод поразрядного умножения, что называется «в столбик».

Что сначала – умножение или сложение?

Любое математическое выражение – это фактически запись учетчика «с полей» о результатах каких-либо действий. Допустим, сбора урожая помидоров:

  • 5 взрослых работников собрали по 500 помидоров каждый и выполнили норму.
  • 2 школьников не ходили на уроки математики и помогали взрослым: собрали по 50 помидоров, норму не выполнили, съели 30 помидоров, надкусили и испортили еще 60 помидоров, 70 помидоров было изъято из карманов помощников. Зачем брали с собой их в поле – непонятно.

Все помидоры сдавали учетчику, он укладывал их по кучкам.

Запишем результат «сбора» урожая в виде выражения:

  • 500 + 500 + 500 + 500 + 500 – это кучки взрослых работников;
  • 50 + 50 – это кучки малолетних работников;
  • 70 – изъято из карманов школьников (испорченное и надкусанное в зачет результата не идет).

Получаем пример для школы, запись учетчика результатов работы:

500 + 500 +500 +500 +500 + 50 +50 + 70 =?;

Здесь можно применить группировку: 5 кучек по 500 помидоров − это можно записать через операцию умножения: 5 ∙ 500.

Две кучки по 50 – это тоже можно записать через умножение.

И одна кучка 70 помидоров.

5 ∙ 500 + 2 ∙ 50 + 1 ∙ 70 =?

И что делать в примере сначала − умножение или сложение? Так вот, складывать можно только помидоры. Нельзя сложить 500 помидоров и 2 кучки. Они не складываются. Поэтому сначала нужно всегда все записи привести к базовым операциям сложения, то есть в первую очередь вычислить все операции группировки-умножения. Совсем простыми словами – сначала выполняется умножение, а сложение уже потом. Если умножить 5 кучек по 500 помидоров каждая, то получится 2500 помидоров. А дальше их уже можно складывать с помидорами из других кучек.

2500 + 100 + 70 = 2 670

При изучении ребенком математики нужно донести до него, что это инструмент, используемый в повседневной жизни. Математические выражения являются, по сути (в самом простом варианте начальной школы), складскими записями о количестве товаров, денег (очень легко воспринимается школьниками), других предметов.

Соответственно, любое произведение – это сумма содержимого некоторого количества одинаковых емкостей, ящиков, кучек, содержащих одинаковое количество предметов. И что сначала умножение, а сложение потом, то есть сначала начала вычислить общее количество предметов, а затем уже складывать их между собой.

Деление

Операция деления отдельно не рассматривается, она обратная умножению. Нужно что-то распределить по коробкам, так, чтобы во всех коробках было одинаковое заданное количество предметов. Самый прямой аналог в жизни – это фасовка.

Скобки

Большое значение в решении примеров имеют скобки. Скобки в арифметике – математический знак, используемый для регулирования последовательности вычислений в выражении (примере).

Умножение и деление имеют приоритет выше, чем сложение и вычитание. А скобки имеют приоритет выше, чем умножение и деление.

Все, что записано в скобках, вычисляется в первую очередь. Если скобки вложенные, то сначала вычисляется выражение во внутренних скобках. И это непреложное правило. Как только выражение в скобках вычислено, скобки пропадают, а на их месте возникает число. Варианты раскрытия скобок с неизвестными здесь не рассматриваются. Так делают до тех пор, пока все они не исчезнут из выражения.

  • Это как коробочки с конфетами в большом мешке. Сначала нужно раскрыть все коробочки и ссыпать в большой мешок: (25 – 5 ) = 20. Пять конфет из коробочки сразу заслали отличнице Люде, которая приболела и в празднике не участвует. Остальные конфеты − в мешок!
  • Потом связать конфеты в пучки по 5 штук: 20 : 5 = 4.
  • Потом добавить в мешок еще 2 пучка конфет, чтобы можно было поделить на троих детей без драки. Признаки деления на 3 в данной статье не рассматриваются.
  • (20 : 5 + 2) : 3 = (4 +2) : 3 = 6 : 3 = 2

    Итого: трем детям по два пучка конфет (по пучку в руку), по 5 конфет в пучке.

    Если вычислить первые скобки в выражении и переписать все заново, пример станет короче. Метод не быстрый, с большим расходом бумаги, зато удивительно эффективный. Заодно тренирует внимательность при переписывании. Пример приводится к виду, когда остается только один вопрос, сначала умножение или сложение без скобок. То есть к такому виду, когда скобок уже и нет. Но ответ на этот вопрос уже есть, и нет смысла обсуждать, что идет сначала – умножение или сложение.

    «Вишенка на торте»

    И напоследок. К математическому выражению не применимы правила русского языка – читать и выполнять слева направо:

    Это простенький пример может довести до истерики ребенка или испортить вечер его маме. Потому что именной ей придется объяснять второкласснику, что бывают отрицательные числа. Или рушить авторитет «МарьиВановны», которая сказала, что: «Нужно слева направо и по порядку».

    «Совсем вишня»

    В Сети гуляет пример, вызывающий затруднения у взрослых дяденек и тетенек. Он не совсем по рассматриваемой теме, что сначала – умножение или сложение. Он вроде как про то, что сначала выполняете действие в скобках.

    От перестановки слагаемых сумма не изменяется, от перестановки множителей тоже. Нужно просто записывать выражение так, чтобы не было потом мучительно стыдно.

    6 : 2 ∙ (1+2) = 6 ∙ ½ ∙ (1+2) = 6 ∙ ½ ∙ 3 = 3 ∙ 3 = 9

    На данном уроке подробно рассмотрен порядок выполнения арифметических действий в выражениях без скобок и со скобками. Учащимся предоставляется возможность в ходе выполнения заданий определить, зависит ли значение выражений от порядка выполнения арифметических действий, узнать отличается ли порядок арифметических действий в выражениях без скобок и со скобками, потренироваться в применении изученного правила, найти и исправить ошибки, допущенные при определении порядка действий.

    Наблюдение за изменением значения выражения от порядка выполнения арифметических действий

    В жизни мы постоянно выполняем какие-либо действия: гуляем, учимся, читаем, пишем, считаем, улыбаемся, ссоримся и миримся. Эти действия мы выполняем в разном порядке. Иногда их можно поменять местами, а иногда нет. Например, собираясь утром в школу, можно сначала сделать зарядку, затем заправить постель, а можно наоборот. Но нельзя сначала уйти в школу, а потом надеть одежду.

    А в математике обязательно ли выполнять арифметические действия в определенном порядке?

    Сравним выражения:
    8-3+4 и 8-3+4

    Видим, что оба выражения совершенно одинаковы.

    Выполним действия в одном выражения слева направо, а в другом справа налево. Числами можно проставить порядок выполнения действий (рис. 1).

    Рис. 1. Порядок действий

    В первом выражении мы сначала выполним действие вычитания, а затем к результату прибавим число 4.

    Во втором выражении сначала найдем значение суммы, а потом из 8 вычтем полученный результат 7.

    Видим, что значения выражений получаются разные.

    Сделаем вывод: порядок выполнения арифметических действий менять нельзя.

    Порядок выполнения арифметических действий в выражениях без скобок

    Узнаем правило выполнения арифметических действий в выражениях без скобок.

    Если в выражение без скобок входят только сложение и вычитание или только умножение и деление, то действия выполняют в том порядке, в каком они написаны.

    В этом выражении имеются только действия сложения и вычитания. Эти действия называют действиями первой ступени.

    Выполняем действия слева направо по порядку (рис. 2).

    Рис. 2. Порядок действий

    Рассмотрим второе выражение

    В этом выражении имеются только действия умножения и деления – это действия второй ступени.

    Выполняем действия слева направо по порядку (рис. 3).

    Рис. 3. Порядок действий

    В каком порядке выполняются арифметические действия, если в выражении имеются не только действия сложения и вычитания, но и умножения и деления?

    Если в выражение без скобок входят не только действия сложения и вычитания, но и умножения и деления, или оба этих действия, то сначала выполняют по порядку (слева направо) умножение и деление, а затем сложение и вычитание.

    Рассуждаем так. В этом выражении имеются действия сложения и вычитания, умножения и деления. Действуем по правилу. Сначала выполняем по порядку (слева направо) умножение и деление, а затем сложение и вычитание. Расставим порядок действий.

    Вычислим значение выражения.

    Порядок выполнения арифметических действий в выражениях со скобками

    В каком порядке выполняются арифметические действия, если в выражении имеются скобки?

    Если в выражении имеются скобки, то сначала вычисляют значение выражений в скобках.

    Мы видим, что в этом выражении имеется действие в скобках, значит, это действие выполним первым, затем по порядку умножение и сложение. Расставим порядок действий.

    Вычислим значение выражения.

    Правило выполнения арифметических действий в выражениях без скобок и со скобками

    Как нужно рассуждать, чтобы правильно установить порядок арифметических действий в числовом выражении?

    Прежде чем приступить к вычислениям, надо рассмотреть выражение (выяснить, есть ли в нём скобки, какие действия в нём имеются) и только после этого выполнять действия в следующем порядке:

    1. действия, записанные в скобках;

    2. умножение и деление;

    3. сложение и вычитание.

    Схема поможет запомнить это несложное правило (рис. 4).

    Рис. 4. Порядок действий

    Выполнение тренировочных заданий на изученное правило

    Рассмотрим выражения, установим порядок действий и выполним вычисления.

    Будем действовать по правилу. В выражении 43 – (20 – 7) +15 имеются действия в скобках, а также действия сложения и вычитания. Установим порядок действий. Первым действием выполним действие в скобках, а затем по порядку слева направо вычитание и сложение.

    43 – (20 – 7) +15 =43 – 13 +15 = 30 + 15 = 45

    В выражении 32 + 9 * (19 – 16) имеются действия в скобках, а также действия умножения и сложения. По правилу первым выполним действие в скобках, затем умножение (число 9 умножаем на результат, полученный при вычитании) и сложение.

    32 + 9 * (19 – 16) =32 + 9 * 3 = 32 + 27 = 59

    В выражении 2*9-18:3 отсутствуют скобки, зато имеются действия умножения, деления и вычитания. Действуем по правилу. Сначала выполним слева направо умножение и деление, а затем от результата, полученного при умножении, вычтем результат, полученный при делении. То есть первое действие – умножение, второе – деление, третье – вычитание.

    Узнаем, правильно ли определен порядок действий в следующих выражениях.

    В этом выражении скобки отсутствуют, значит, сначала выполняем слева направо умножение или деление, затем сложение или вычитание. В данном выражении первое действие – деление, второе – умножение. Третье действие должно быть сложение, четвертое – вычитание. Вывод: порядок действий определен верно.

    Найдем значение данного выражения.

    Во втором выражении имеются скобки, значит, сначала выполняем действие в скобках, затем слева направо умножение или деление, сложение или вычитание. Проверяем: первое действие – в скобках, второе – деление, третье – сложение. Вывод: порядок действий определен неверно. Исправим ошибки, найдем значение выражения.

    В этом выражении также имеются скобки, значит, сначала выполняем действие в скобках, затем слева направо умножение или деление, сложение или вычитание. Проверяем: первое действие – в скобках, второе – умножение, третье – вычитание. Вывод: порядок действий определен неверно. Исправим ошибки, найдем значение выражения.

    Расставим порядок действий в выражении, используя изученное правило (рис. 5).

    Рис. 5. Порядок действий

    Мы не видим числовых значений, поэтому не сможем найти значение выражений, однако потренируемся применять изученное правило.

    Действуем по алгоритму.

    В первом выражении имеются скобки, значит, первое действие в скобках. Затем слева направо умножение и деление, потом слева направо вычитание и сложение.

    Во втором выражении также имеются скобки, значит, первое действие выполняем в скобках. После этого слева направо умножение и деление, после этого – вычитание.

    Проверим себя (рис. 6).

    Рис. 6. Порядок действий

    Сегодня на уроке мы познакомились с правилом порядка выполнения действий в выражениях без скобок и со скобками.

    Список литературы

    1. М.И. Моро, М.А. Бантова и др. Математика: Учебник. 3 класс: в 2-х частях, часть 1. – М.: «Просвещение», 2012.
    2. М.И. Моро, М.А. Бантова и др. Математика: Учебник. 3 класс: в 2-х частях, часть 2. – М.: «Просвещение», 2012.
    3. М.И. Моро. Уроки математики: Методические рекомендации для учителя. 3 класс. – М.: Просвещение, 2012.
    4. Нормативно-правовой документ. Контроль и оценка результатов обучения. – М.: «Просвещение», 2011.
    5. «Школа России»: Программы для начальной школы. – М.: «Просвещение», 2011.
    6. С.И. Волкова. Математика: Проверочные работы. 3 класс. – М.: Просвещение, 2012.
    7. В.Н. Рудницкая. Тесты. – М.: «Экзамен», 2012.

    Дополнительные рекомендованные ссылки на ресурсы сети Интернет

    Домашнее задание

    1. Определи порядок действий в данных выражениях. Найди значение выражений.

    2. Определи, в каком выражении такой порядок выполнения действий:

    1. умножение; 2. деление;. 3. сложение; 4. вычитание; 5. сложение. Найди значение данного выражения.

    3. Составь три выражения, в которых такой порядок выполнения действий:

    1. умножение; 2. сложение; 3. вычитание

    1. сложение; 2. вычитание; 3. сложение

    1. умножение; 2. деление; 3. сложение

    Найди значение этих выражений.

    Если вы нашли ошибку или неработающую ссылку, пожалуйста, сообщите нам – сделайте свой вклад в развитие проекта.

    В первую очередь умножение или сложение. Порядок выполнения действий, правила, примеры

    При расчётах примеров нужно соблюдать определённый порядок действий. С помощью правил ниже, мы разберёмся в каком порядке выполняются действия и для чего нужны скобки.

    Если в выражении скобок нет, то:

  • сначала выполняем слева направо все действия умножения и деления;
  • а потом слева направо все действия сложения и вычитания.
  • Рассмотрим порядок действий в следующем примере.

    Напоминаем вам, что порядок действий в математике расставляется слева направо (от начала к концу примера).

    При вычислении значения выражения можно вести запись двумя способами.

    Первый способ

    • Каждое действие записывается отдельно со своим номером под примером.
    • После выполнения последнего действия ответ обязательно записывается в исходный пример.
    • При расчёте результатов действий с двузначными и/или трёхзначными числами обязательно приводите свои расчёты в столбик.

      Второй способ

    • Второй способ называется запись «цепочкой». Все вычисления проводятся в точно таком же порядке действий, но результаты записываются сразу после знака равно.
    • Если выражение содержит скобки, то сначала выполняют действия в скобках.

      Внутри самих скобок действует правило порядка действий как в выражениях без скобок.

      Если внутри скобок находятся ещё одни скобки, то сначала выполняются действия внутри вложенных (внутренних) скобок.

      Порядок действий и возведение в степень

      Если в примере содержится числовое или буквенное выражение в скобках, которое надо возвести в степень, то:

      • Сначала выполняем все действия внутри скобок
      • Затем возводим в степень все скобки и числа, стоящие в степени, слева направо (от начала к концу примера).
      • Выполняем оставшиеся действия в обычном порядке
      • Порядок выполнения действий, правила, примеры.

        Числовые,буквенные выражения и выражения с переменными в своей записи могут содержать знаки различных арифметических действий. При преобразовании выражений и вычислении значений выражений действия выполняются в определенной очередности, иными словами, нужно соблюдать порядок выполнения действий .

        В этой статье мы разберемся, какие действия следует выполнять сначала, а какие следом за ними. Начнем с самых простых случаев, когда выражение содержит лишь числа или переменные, соединенные знаками плюс, минус, умножить и разделить. Дальше разъясним, какого порядка выполнения действий следует придерживаться в выражениях со скобками. Наконец, рассмотрим, в какой последовательности выполняются действия в выражениях, содержащих степени, корни и другие функции.

        Навигация по странице.

        Сначала умножение и деление, затем сложение и вычитание

        В школе дается следующее правило, определяющее порядок выполнения действий в выражениях без скобок :

        • действия выполняются по порядку слева направо,
        • причем сначала выполняется умножение и деление, а затем – сложение и вычитание.
        • Озвученное правило воспринимается достаточно естественно. Выполнение действий по порядку слева направо объясняется тем, что у нас принято вести записи слева направо. А то, что умножение и деление выполняется перед сложением и вычитанием объясняется смыслом, который в себе несут эти действия.

          Рассмотрим несколько примеров применения этого правила. Для примеров будем брать простейшие числовые выражения, чтобы не отвлекаться на вычисления, а сосредоточиться именно на порядке выполнения действий.

          Выполните действия 7−3+6 .

          Исходное выражение не содержит скобок, а также оно не содержит умножения и деления. Поэтому нам следует выполнить все действия по порядку слева направо, то есть, сначала мы от 7 отнимаем 3 , получаем 4 , после чего к полученной разности 4 прибавляем 6 , получаем 10 .

          Кратко решение можно записать так: 7−3+6=4+6=10 .

          Укажите порядок выполнения действий в выражении 6:2·8:3 .

          Чтобы ответить на вопрос задачи, обратимся к правилу, указывающему порядок выполнения действий в выражениях без скобок. В исходном выражении содержатся лишь действия умножения и деления, а согласно правилу, их нужно выполнять по порядку слева направо.

          сначала 6 делим на 2 , это частное умножаем на 8 , наконец, полученный результат делим на 3.

          Вычислите значение выражения 17−5·6:3−2+4:2 .

          Сначала определим, в каком порядке следует выполнять действия в исходном выражении. Оно содержит и умножение с делением, и сложение с вычитанием. Сначала слева направо нужно выполнить умножение и деление. Так 5 умножаем на 6 , получаем 30 , это число делим на 3 , получаем 10 . Теперь 4 делим на 2 , получаем 2 . Подставляем в исходное выражение вместо 5·6:3 найденное значение 10 , а вместо 4:2 — значение 2 , имеем 17−5·6:3−2+4:2=17−10−2+2 .

          В полученном выражении уже нет умножения и деления, поэтому остается по порядку слева направо выполнить оставшиеся действия: 17−10−2+2=7−2+2=5+2=7 .

          На первых порах, чтобы не перепутать порядок выполнения действий при вычислении значения выражения, удобно над знаками действий расставить цифры, соответствующие порядку их выполнения. Для предыдущего примера это выглядело бы так: .

          Этого же порядка выполнения действий – сначала умножение и деление, затем сложение и вычитание — следует придерживаться и при работе с буквенными выражениями.

          Действия первой и второй ступени

          В некоторых учебниках по математике встречается разделение арифметических действий на действия первой и второй ступени. Разберемся с этим.

          Действиями первой ступени называют сложение и вычитание, а умножение и деление называют действиями второй ступени .

          В этих терминах правило из предыдущего пункта, определяющее порядок выполнения действий, запишется так: если выражение не содержит скобок, то по порядку слева направо сначала выполняются действия второй ступени (умножение и деление), затем – действия первой ступени (сложение и вычитание).

          Порядок выполнения арифметических действий в выражениях со скобками

          Выражения часто содержат скобки, указывающие порядок выполнения действий. В этом случае правило, задающее порядок выполнения действий в выражениях со скобками , формулируется так: сначала выполняются действия в скобках, при этом также по порядку слева направо выполняется умножение и деление, затем – сложение и вычитание.

          Итак, выражения в скобках рассматриваются как составные части исходного выражения, и в них сохраняется уже известный нам порядок выполнения действий. Рассмотрим решения примеров для большей ясности.

          Выполните указанные действия 5+(7−2·3)·(6−4):2 .

          Выражение содержит скобки, поэтому сначала выполним действия в выражениях, заключенных в эти скобки. Начнем с выражения 7−2·3 . В нем нужно сначала выполнить умножение, и только потом вычитание, имеем 7−2·3=7−6=1 . Переходим ко второму выражению в скобках 6−4 . Здесь лишь одно действие – вычитание, выполняем его 6−4=2 .

          Подставляем полученные значения в исходное выражение: 5+(7−2·3)·(6−4):2=5+1·2:2 . В полученном выражении сначала выполняем слева направо умножение и деление, затем – вычитание, получаем 5+1·2:2=5+2:2=5+1=6 . На этом все действия выполнены, мы придерживались такого порядка их выполнения: 5+(7−2·3)·(6−4):2 .

          Запишем краткое решение: 5+(7−2·3)·(6−4):2=5+1·2:2=5+1=6 .

          Бывает, что выражение содержит скобки в скобках. Этого бояться не стоит, нужно лишь последовательно применять озвученное правило выполнения действий в выражениях со скобками. Покажем решение примера.

          Выполните действия в выражении 4+(3+1+4·(2+3)) .

          Это выражение со скобками, это означает, что выполнение действий нужно начинать с выражения в скобках, то есть, с 3+1+4·(2+3) . Это выражение также содержит скобки, поэтому нужно сначала выполнить действия в них. Сделаем это: 2+3=5 . Подставив найденное значение, получаем 3+1+4·5 . В этом выражении сначала выполняем умножение, затем – сложение, имеем 3+1+4·5=3+1+20=24 . Исходное значение, после подстановки этого значения, принимает вид 4+24 , и остается лишь закончить выполнение действий: 4+24=28 .

          Вообще, когда в выражении присутствуют скобки в скобках, то часто бывает удобно выполнение действий начинать с внутренних скобок и продвигаться к внешним.

          Например, пусть нам нужно выполнить действия в выражении (4+(4+(4−6:2))−1)−1 . Сначала выполняем действия во внутренних скобках, так как 4−6:2=4−3=1 , то после этого исходное выражение примет вид (4+(4+1)−1)−1 . Опять выполняем действие во внутренних скобках, так как 4+1=5 , то приходим к следующему выражению (4+5−1)−1 . Опять выполняем действия в скобках: 4+5−1=8 , при этом приходим к разности 8−1 , которая равна 7 .

          Порядок выполнения действий в выражениях с корнями, степенями, логарифмами и другими функциями

          Если в выражение входят степени, корни, логарифмы, синус, косинус, тангенс и котангенс, а также другие функции, то их значения вычисляются до выполнения остальных действий, при этом также учитываются правила из предыдущих пунктов, задающие порядок выполнения действий. Иными словами, перечисленные вещи, грубо говоря, можно считать заключенными в скобки, а мы знаем, что сначала выполняются действия в скобках.

          Рассмотрим решения примеров.

          Выполните действия в выражении (3+1)·2+6 2:3−7 .

          В этом выражении содержится степень 6 2 , ее значение нужно вычислить до выполнения остальных действий. Итак, выполняем возведение в степень: 6 2 =36 . Подставляем это значение в исходное выражение, оно примет вид (3+1)·2+36:3−7 .

          Дальше все понятно: выполняем действия в скобках, после чего остается выражение без скобок, в котором по порядку слева направо сначала выполняем умножение и деление, а затем – сложение и вычитание. Имеем (3+1)·2+36:3−7=4·2+36:3−7= 8+12−7=13 .

          Другие, в том числе и более сложные примеры выполнения действий в выражениях с корнями, степенями и т.п., Вы можете посмотреть в статье вычисление значений выражений.

          cleverstudents.ru

          Онлайн игры,тренажеры,презентации,уроки,энциклопедии,статьи

          Post navigation

          Примеры со скобками, урок с тренажерами.

          Мы рассмотрим в этой статье три варианта примеров:

          1. Примеры со скобками (действия сложения и вычитания)

          2. Примеры со скобками (сложение, вычитание, умножение, деление)

          3. Примеры, в которых много действий

          1 Примеры со скобками (действия сложения и вычитания)

          Рассмотрим три примера. В каждом из них порядок действий обозначен цифрами красного цвета:

          Мы видим, что порядок действий в каждом примере будет разный, хотя числа и знаки одинаковые. Это происходит потому, что во втором и третьем примере есть скобки.

        • Если в примере нет скобок , мы выполняем все действия по порядку, слева направо.
        • Если в примере есть скобки , то сначала мы выполняем действия в скобках, и лишь потом все остальные действия, начиная слева направо.
        • *Это правило для примеров без умножения и деления. Правила для примеров со скобками, включающих действия умножения и деления мы рассмотрим во второй части этой статьи.

          Чтобы не запутаться в примере со скобками, можно превратить его в обычный пример, без скобок. Для этого результат, полученный в скобках, записываем над скобками, далее переписываем весь пример, записывая вместо скобок этот результат, и далее выполняем все действия по порядку, слева направо:

          В несложных примерах можно все эти операции производить в уме. Главное — сначала выполнить действие в скобках и запомнить результат, а затем считать по порядку, слева направо.

          А теперь — тренажеры!

          1) Примеры со скобками в пределах до 20. Онлайн тренажер.

          2) Примеры со скобками в пределах до 100. Онлайн тренажер.

          3) Примеры со скобками. Тренажер №2

          4) Вставь пропущенное число — примеры со скобками. Тренажер

          2 Примеры со скобками (сложение, вычитание, умножение, деление)

          Теперь рассмотрим примеры, в которых кроме сложения и вычитания есть умножение и деление.

          Сначала рассмотрим примеры без скобок:

        • Если в примере нет скобок , сначала выполняем действия умножения и деления по порядку, слева направо. Затем — действия сложения и вычитания по порядку, слева направо.
        • Если в примере есть скобки , то сначала мы выполняем действия в скобках, затем умножение и деление, и затем — сложение и вычитание начиная слева направо.
        • Есть одна хитрость, как не запутаться при решении примеров на порядок действий. Если нет скобок, то выполняем действия умножения и деления, далее переписываем пример, записывая вместо этих действий полученные результаты. Затем выполняем сложение и вычитание по порядку:

          Если в примере есть скобки, то сначала нужно избавиться от скобок: переписать пример, записывая вместо скобок полученный в них результат. Затем нужно выделить мысленно части примера, разделенные знаками «+» и «-«, и посчитать каждую часть отдельно. Затем выполнить сложение и вычитание по порядку:

          3 Примеры, в которых много действий

          Если в примере много действий, то удобнее будет не расставлять порядок действий во всем примере, а выделить блоки, и решить каждый блок отдельно. Для этого находим свободные знаки «+» и «–» (свободные — значит не в скобках, на рисунке показаны стрелочками).

          Эти знаки и будут делить наш пример на блоки:

          Выполняя действия в каждом блоке не забываем про порядок действий, приведенный выше в статье. Решив каждый блок, выполняем действия сложения и вычитания по порядку.

          А теперь закрепляем решение примеров на порядок действий на тренажерах!

          1. Примеры со скобками в пределах чисел до 100, действия сложения, вычитания, умножения и деления. Онлайн тренажер.

          2. Тренажер по математике 2 — 3 класс «Расставь порядок действий (буквенные выражения).»

          3. Порядок действий (расставляем порядок и решаем примеры)

          Порядок действий в математике 4 класс

          Начальная школа подходит к концу, скоро ребёнок шагнёт в углубленный мир математики. Но уже в этот период школьник сталкивается с трудностями науки. Выполняя простое задание, ребёнок путается, теряется, что в результате приводит к отрицательной отметке за выполненную работу. Чтобы избежать подобных неприятностей, нужно при решении примеров, уметь ориентироваться в порядке, по которому нужно решать пример. Не верно распределив действия, ребёнок не правильно выполняет задание. В статье раскрываются основные правила решения примеров, содержащих в себе весь спектр математических вычислений, включая скобки. Порядок действий в математике 4 класс правила и примеры.

          Перед выполнением задания попросите своё чадо пронумеровать действия, которые он собирается выполнить. Если возникли затруднения – помогите.

          Некоторые правила, которые необходимо соблюдать при решении примеров без скобок:

          Если в задании необходимо выполнить ряд действий, нужно сначала выполнить деление или умножение, затем сложение. Все действия выполняются по ходу письма. В противном случае, результат решения будет не верным.

          Если в примере требуется выполнить сложение и вычитание, выполняем по порядку, слева направо.

          27-5+15=37 (при решении примера руководствуемся правилом. Сначала выполняем вычитание, затем – сложение).

          Научите ребёнка всегда планировать и нумеровать выполняемые действия.

          Ответы на каждое решённое действие записываются над примером. Так ребёнку гораздо легче будет ориентироваться в действиях.

          Рассмотрим ещё один вариант, где необходимо распределить действия по порядку:

          Как видим, при решении соблюдено правило, сначала ищем произведение, после — разность.

          Это простые примеры, при решении которых, необходима внимательность. Многие дети впадают в ступор при виде задания, в котором присутствует не только умножение и деление, но и скобки. У школьника, не знающего порядок выполнения действий, возникают вопросы, которые мешают выполнить задание.

          Как говорилось в правиле, сначала найдём произведение или частное, а потом всё остальное. Но тут же есть скобки! Как поступить в этом случае?

          Решение примеров со скобками

          Разберём конкретный пример:

        • При выполнении данного задания, сначала найдём значение выражения, заключённого в скобки.
        • Начать следует с умножения, далее – сложение.
        • После того, как выражение в скобках решено, приступаем к действиям вне их.
        • По правилам порядка действий, следующим шагом будет умножение.
        • Завершающим этапом станет вычитание.
        • Как видим на наглядном примере, все действия пронумерованы. Для закрепления темы предложите ребёнку решить самостоятельно несколько примеров:

          Порядок, по которому следует вычислять значение выражения уже расставлен. Ребёнку останется только выполнить непосредственно решение.

          Усложним задачу. Пусть ребёнок найдёт значение выражений самостоятельно.

          7*3-5*4+(20-19) 14+2*3-(13-9)
          17+2*5+(28-2) 5*3+15-(2-1*2)
          24-3*2-(56-4*3) 14+12-3*(21-7)

          Приучите ребёнка решать все задания в черновом варианте. В таком случае, у школьника будет возможность исправить не верное решение или помарки. В рабочей тетради исправления не допустимы. Выполняя самостоятельно задания, дети видят свои ошибки.

          Родители, в свою очередь, должны обратить внимание на ошибки, помочь ребёнку разобраться и исправить их. Не стоит нагружать мозг школьника большими объёмами заданий. Такими действиями вы отобьёте стремление ребёнка к знаниям. Во всём должно быть чувство меры.

          Делайте перерыв. Ребёнок должен отвлекаться и отдыхать от занятий. Главное помнить, что не все обладают математическим складом ума. Может из вашего ребёнка вырастет знаменитый философ.

          detskoerazvitie.info

          Урок по математике 2 класс Порядок действий в выражениях со скобками.

          Успейте воспользоваться скидками до 50% на курсы «Инфоурок»

          Цель: 1.

          2.

          3. Закрепить знание таблицы умножения и деления на 2 – 6, понятия делителя и

          4. Учить работать в парах с целью развития коммуникативных качеств.

          Оборудование * : + — (), геометрический материал.

          Раз, два – выше голова.

          Три, четыре – руки шире.

          Пять, шесть – всем присесть.

          Семь, восемь – лень отбросим.

          Но сначала придется узнать его название. Для этого нужно выполнить несколько заданий:

          6 + 6 + 6 … 6 * 4 6 * 4 + 6… 6 * 5 – 6 14 дм 5 см… 4 дм 5 см

          Пока мы вспоминали о порядке действий в выражениях, с замком происходили чудеса. Мы были только что у ворот, а теперь попали в коридор. Смотрите, дверь. А на ней замок. Откроем?

          1. Из числа 20 вычесть частное чисел 8 и 2.

          2. Разность чисел 20 и 8 разделить на 2.

          — Чем отличаются результаты?

          — Кто сможет назвать тему нашего урока?

          (на массажных ковриках)

          По дорожке, по дорожке

          Скачем мы на правой ножке,

          Скачем мы на левой ножке.

          По тропинке побежим,

          Наше предположение было полностью правильно7

          Где выполняются действия сначала, если в выражении есть скобки?

          Смотрите перед нами «живые примеры». Давайте «оживим» их.

          * : + — ().

          m – c * (a + d) + x

          k: b + (a – c) * t

          6. Работа в парах.

          Для их решения вам понадобиться геометрический материал.

          Учащиеся выполняют задания в парах. После выполнения проверка работы пар у доски.

          Что нового вы узнали?

          8. Домашнее задание.

          Тема: Порядок действий в выражениях со скобками.

          Цель: 1. Вывести правило порядка действий в выражениях со скобками, содержащих все

          4 арифметических действия,

          2. Формировать способность к практическому применению правила,

          4.Учить работать в парах с целью развития коммуникативных качеств.

          Оборудование : учебник, тетради, карточки со знаками действий * : + — (), геометрический материал.

          1 .Физминутка.

          Девять, десять – тихо сесть.

          2. Актуализация опорных знаний.

          Сегодня мы с вами отправляемся в очередное путешествие по стране Знаний городу математика. Нам предстоит посетить один дворец. Что-то я забыла его название. Но не будем расстраиваться, вы сами сможете мне подсказать его название. Пока я переживала, мы подошли к воротам дворца. Войдем?

          1. Сравните выражения:

          2. Расшифруй слово.

          3. Постановка проблемы. Открытие нового.

          Так как же называется дворец?

          А когда в математике мы говорим о порядке?

          Что вы уже знаете о порядке выполнения действий в выражениях?

          — Интересно, нам предлагают записать и решить выражения (учитель читает выражения, учащиеся записывают их и решают).

          20 – 8: 2

          (20 – 8) : 2

          Молодцы. А что интересного в этих выражениях?

          Посмотрите на выражения и их результаты.

          — Что общего в записи выражений?

          — Как вы думаете, почему получились разные результаты, ведь числа были одинаковые?

          Кто рискнет сформулировать правило выполнения действий в выражениях со скобками?

          Правильность этого ответа мы сможем проверить в другой комнате. Отправляемся туда.

          4. Физминутка.

          И по этой же дорожке

          До горы мы добежим.

          Стоп. Немножко отдохнем

          И опять пешком пойдем.

          5. Первичное закрепление изученного.

          Вот мы и пришли.

          Нам нужно решить еще два выражения, чтобы проверить правильность нашего предположения.

          6 * (33 – 25) 54: (6 + 3) 25 – 5 * (9 – 5) : 2

          Для проверки правильности предположения откроем учебники на стр. 33 и прочитаем правило.

          Как нужно выполнять действия после решения в скобках?

          На доске написаны буквенные выражения и лежат карточки со знаками действий * : + — (). Дети выходят к доске по одному, берут карточку с тем действием, которое нужно сделать сначала, потом выходит второй ученик и берет карточку со вторым действием и т. д.

          а + (а –в)

          а * (в +с) : d t

          m c * ( a + d ) + x

          k : b + ( a c ) * t

          (a – b) : t + d

          6. Работа в парах. Автономная некоммерческая организация Бюро судебных экспертиз Судебная экспертиза. Несудебная экспертиза Рецензия на экспертизу. Оценка Автономная некоммерческая организация «Бюро судебных экспертиз» в Москве – центр […]

        • Особенности бухгалтерского учета субсидий Государство стремится поддержать малое и среднее предпринимательство. Такая поддержка наиболее часто выражается в форме предоставления субсидий – безвозмездных выплат из […]
        • Жалоба на педиатра Жалоба на педиатра — официальный документ, устанавливающий требования пациента и описывающий суть возникновения таких требований. Согласно статье 4 Федерального закона «О порядке рассмотрения […]
        • Ходатайство об уменьшении размера исковых требований Один из видов уточнения иска — ходатайство об уменьшении размера исковых требований. Когда истец неправильно определил цену иска. Или ответчик частично исполнил […]
        • Черный рынок доллара в Киеве Валютный аукцион по покупке доллара в Киеве Внимание: администрация не несёт ответственности за содержание объявлений на валютном аукционе. Правила публикации объявлений на валютном […]

    Видеоурок «Порядок выполнения действий» подробно поясняет важную тему математики — последовательность выполнения арифметических операций при решении выражения. В ходе видеоурока рассматривается, какой приоритет имеют различные математические операции, как это применяется в вычислении выражений, приводятся примеры для усвоения материала, обобщаются полученные знания в решении заданий, где имеются все рассмотренные операции. С помощью видеоурока учитель имеет возможность быстрее достичь целей урока, повысить его эффективность. Видео может применяться в качестве наглядного материала, сопровождающего объяснение учителя, а также в качестве самостоятельной части урока.

    В наглядном материале используются приемы, которые помогают лучше достичь понимания темы, а также запомнить важные правила. С помощью цвета и разного написания выделяются особенности и свойства операций, отмечаются особенности решения примеров. Анимационные эффекты помогают подавать последовательно учебный материал, а также обратить внимание учеников на важные моменты. Видео озвучено, поэтому дополняется комментариями учителя, помогающими ученику понять и запомнить тему.

    Видеоурок начинается с представления темы. Затем отмечается, что умножение, вычитание являются операциями первой ступени, операции умножения и деления названы операциями второй ступени. Данным определением нужно будет оперировать дальше, выведено на экран и выделено цветным крупным шрифтом. Затем представляются правила, составляющие порядок выполнения операций. Выводится первое правило порядка, которое указывает, что при отсутствии скобок в выражении, наличию действий одной ступени, данные действия необходимо производить по порядку. Во втором правиле порядка утверждается, что при наличии действий обеих ступеней и отсутствии скобок, производятся первыми операции второй ступени, потом производятся операции первой ступени. Третье правило устанавливает порядок выполнения операций, для выражений, включающих скобки. Отмечается, что в этом случае сначала производятся операции в скобках. Формулировки правил выделены цветным шрифтом и рекомендованы к запоминанию.

    Далее предлагается усвоить порядок выполнения операций, рассматривая примеры. Описывается решение выражения с содержанием только операций сложения, вычитания. Отмечаются основные особенности, которые влияют на порядок вычислений — отсутствуют скобки, присутствуют операции первой ступени. Ниже расписано по действиям, как выполняются вычисления, сначала вычитание, затем два раза сложение, а затем вычитание.

    Во втором примере 780:39·212:156·13 требуется вычислить выражение, выполняя действия согласно порядку. Отмечается, что в данном выражении содержатся исключительно операции второй ступени, без скобок. В данном примере все действия производятся строго слева направо. Ниже поочередно расписываются действия, постепенно подходя к ответу. В результате вычисления получается число 520.

    В третьем примере рассматривается решение примера, в котором есть операции обеих ступеней. Отмечается, что в данном выражении отсутствуют скобки, но есть действия обеих ступеней. Согласно порядку выполнения операций, производятся операции второй ступени, после этого — операции первой ступени. Ниже — по действиям расписывается решение, в котором выполняются сначала три операции — умножение, деление, еще одно деление. Затем с найденными значениями произведения и частных производятся операции первой ступени. В ходе решения фигурными скобками объединены действия каждой ступени для наглядности.

    В следующем примере содержатся скобки. Поэтому демонстрируется, что первые вычисления производятся над выражениями в скобках. После них производятся операции второй ступени, следом — первой.

    Далее представлено замечание о том, в каких случаях можно не записывать скобки при решении выражений. Замечено, что это возможно только в случае, когда устранение скобок не изменить порядок выполнения операций. Примером служит выражение со скобками (53-12)+14, которое содержит только операции первой ступени. Переписав 53-12+14 с устранением скобок, можно отметить, что порядок поиска значения не изменится — сначала выполняется вычитание 53-12=41, а затем сложение 41+14=55. Ниже отмечается, что менять порядок операций при нахождении решения выражения можно, используя свойства операций.

    В конце видеоурока изученный материал обобщается в выводе, что каждое выражение, требующее решения, задает определенную программу для вычисления, состоящую из команд. Пример такой программы представляется при описании решения сложного примера, представляющего собой частное (814+36·27) и (101-2052:38). Заданная программа содержит пункты: 1) найти произведение 36 с 27, 2) добавить к 814 найденную сумму, 3) поделить на 38 число 2052, 4) отнять из числа 101 результат деления 3 пункта, 5) поделить результат выполнения пункта 2 на результат пункта 4.

    В конце видеоурока представлен перечень вопросов, на которые предлагается ответить ученикам. В их числе умение отличить действия первой и второй ступеней, вопросы о порядке выполнения действий в выражениях с действиями одной ступени и разных ступеней, о порядке выполнения действий при наличии скобок в выражении.

    Видеоурок «Порядок выполнения действий» рекомендуется применять на традиционном школьном уроке для повышения эффективности урока. Также наглядный материал будет полезен для проведения дистанционного обучения. Если ученику необходимо дополнительное занятие для освоения темы или он изучает ее самостоятельно, видео может быть рекомендовано для самостоятельного изучения.

    На данном уроке подробно рассмотрен порядок выполнения арифметических действий в выражениях без скобок и со скобками. Учащимся предоставляется возможность в ходе выполнения заданий определить, зависит ли значение выражений от порядка выполнения арифметических действий, узнать отличается ли порядок арифметических действий в выражениях без скобок и со скобками, потренироваться в применении изученного правила, найти и исправить ошибки, допущенные при определении порядка действий.

    В жизни мы постоянно выполняем какие-либо действия: гуляем, учимся, читаем, пишем, считаем, улыбаемся, ссоримся и миримся. Эти действия мы выполняем в разном порядке. Иногда их можно поменять местами, а иногда нет. Например, собираясь утром в школу, можно сначала сделать зарядку, затем заправить постель, а можно наоборот. Но нельзя сначала уйти в школу, а потом надеть одежду.

    А в математике обязательно ли выполнять арифметические действия в определенном порядке?

    Давайте проверим

    Сравним выражения:
    8-3+4 и 8-3+4

    Видим, что оба выражения совершенно одинаковы.

    Выполним действия в одном выражения слева направо, а в другом справа налево. Числами можно проставить порядок выполнения действий (рис. 1).

    Рис. 1. Порядок действий

    В первом выражении мы сначала выполним действие вычитания, а затем к результату прибавим число 4.

    Во втором выражении сначала найдем значение суммы, а потом из 8 вычтем полученный результат 7.

    Видим, что значения выражений получаются разные.

    Сделаем вывод: порядок выполнения арифметических действий менять нельзя .

    Узнаем правило выполнения арифметических действий в выражениях без скобок.

    Если в выражение без скобок входят только сложение и вычитание или только умножение и деление, то действия выполняют в том порядке, в каком они написаны.

    Потренируемся.

    Рассмотрим выражение

    В этом выражении имеются только действия сложения и вычитания. Эти действия называют действиями первой ступени .

    Выполняем действия слева направо по порядку (рис. 2).

    Рис. 2. Порядок действий

    Рассмотрим второе выражение

    В этом выражении имеются только действия умножения и деления — это действия второй ступени.

    Выполняем действия слева направо по порядку (рис. 3).

    Рис. 3. Порядок действий

    В каком порядке выполняются арифметические действия, если в выражении имеются не только действия сложения и вычитания, но и умножения и деления?

    Если в выражение без скобок входят не только действия сложения и вычитания, но и умножения и деления, или оба этих действия, то сначала выполняют по порядку (слева направо) умножение и деление, а затем сложение и вычитание.

    Рассмотрим выражение.

    Рассуждаем так. В этом выражении имеются действия сложения и вычитания, умножения и деления. Действуем по правилу. Сначала выполняем по порядку (слева направо) умножение и деление, а затем сложение и вычитание. Расставим порядок действий.

    Вычислим значение выражения.

    18:2-2*3+12:3=9-6+4=3+4=7

    В каком порядке выполняются арифметические действия, если в выражении имеются скобки?

    Если в выражении имеются скобки, то сначала вычисляют значение выражений в скобках.

    Рассмотрим выражение.

    30 + 6 * (13 — 9)

    Мы видим, что в этом выражении имеется действие в скобках, значит, это действие выполним первым, затем по порядку умножение и сложение. Расставим порядок действий.

    30 + 6 * (13 — 9)

    Вычислим значение выражения.

    30+6*(13-9)=30+6*4=30+24=54

    Как нужно рассуждать, чтобы правильно установить порядок арифметических действий в числовом выражении?

    Прежде чем приступить к вычислениям, надо рассмотреть выражение (выяснить, есть ли в нём скобки, какие действия в нём имеются) и только после этого выполнять действия в следующем порядке:

    1. действия, записанные в скобках;

    2. умножение и деление;

    3. сложение и вычитание.

    Схема поможет запомнить это несложное правило (рис. 4).

    Рис. 4. Порядок действий

    Потренируемся.

    Рассмотрим выражения, установим порядок действий и выполним вычисления.

    43 — (20 — 7) +15

    32 + 9 * (19 — 16)

    Будем действовать по правилу. В выражении 43 — (20 — 7) +15 имеются действия в скобках, а также действия сложения и вычитания. Установим порядок действий. Первым действием выполним действие в скобках, а затем по порядку слева направо вычитание и сложение.

    43 — (20 — 7) +15 =43 — 13 +15 = 30 + 15 = 45

    В выражении 32 + 9 * (19 — 16) имеются действия в скобках, а также действия умножения и сложения. По правилу первым выполним действие в скобках, затем умножение (число 9 умножаем на результат, полученный при вычитании) и сложение.

    32 + 9 * (19 — 16) =32 + 9 * 3 = 32 + 27 = 59

    В выражении 2*9-18:3 отсутствуют скобки, зато имеются действия умножения, деления и вычитания. Действуем по правилу. Сначала выполним слева направо умножение и деление, а затем от результата, полученного при умножении, вычтем результат, полученный при делении. То есть первое действие — умножение, второе — деление, третье — вычитание.

    2*9-18:3=18-6=12

    Узнаем, правильно ли определен порядок действий в следующих выражениях.

    37 + 9 — 6: 2 * 3 =

    18: (11 — 5) + 47=

    7 * 3 — (16 + 4)=

    Рассуждаем так.

    37 + 9 — 6: 2 * 3 =

    В этом выражении скобки отсутствуют, значит, сначала выполняем слева направо умножение или деление, затем сложение или вычитание. В данном выражении первое действие — деление, второе — умножение. Третье действие должно быть сложение, четвертое — вычитание. Вывод: порядок действий определен верно.

    Найдем значение данного выражения.

    37+9-6:2*3 =37+9-3*3=37+9-9=46-9=37

    Продолжаем рассуждать.

    Во втором выражении имеются скобки, значит, сначала выполняем действие в скобках, затем слева направо умножение или деление, сложение или вычитание. Проверяем: первое действие — в скобках, второе — деление, третье — сложение. Вывод: порядок действий определен неверно. Исправим ошибки, найдем значение выражения.

    18:(11-5)+47=18:6+47=3+47=50

    В этом выражении также имеются скобки, значит, сначала выполняем действие в скобках, затем слева направо умножение или деление, сложение или вычитание. Проверяем: первое действие — в скобках, второе — умножение, третье — вычитание. Вывод: порядок действий определен неверно. Исправим ошибки, найдем значение выражения.

    7*3-(16+4)=7*3-20=21-20=1

    Выполним задание.

    Расставим порядок действий в выражении, используя изученное правило (рис. 5).

    Рис. 5. Порядок действий

    Мы не видим числовых значений, поэтому не сможем найти значение выражений, однако потренируемся применять изученное правило.

    Действуем по алгоритму.

    В первом выражении имеются скобки, значит, первое действие в скобках. Затем слева направо умножение и деление, потом слева направо вычитание и сложение.

    Во втором выражении также имеются скобки, значит, первое действие выполняем в скобках. После этого слева направо умножение и деление, после этого — вычитание.

    Проверим себя (рис. 6).

    Рис. 6. Порядок действий

    Сегодня на уроке мы познакомились с правилом порядка выполнения действий в выражениях без скобок и со скобками.

    Список литературы

    1. М.И. Моро, М.А. Бантова и др. Математика: Учебник. 3 класс: в 2-х частях, часть 1. — М.: «Просвещение», 2012.
    2. М.И. Моро, М.А. Бантова и др. Математика: Учебник. 3 класс: в 2-х частях, часть 2. — М.: «Просвещение», 2012.
    3. М.И. Моро. Уроки математики: Методические рекомендации для учителя. 3 класс. — М.: Просвещение, 2012.
    4. Нормативно-правовой документ. Контроль и оценка результатов обучения. — М.: «Просвещение», 2011.
    5. «Школа России»: Программы для начальной школы. — М.: «Просвещение», 2011.
    6. С.И. Волкова. Математика: Проверочные работы. 3 класс. — М.: Просвещение, 2012.
    7. В.Н. Рудницкая. Тесты. — М.: «Экзамен», 2012.
    1. Festival.1september.ru ().
    2. Sosnovoborsk-soobchestva.ru ().
    3. Openclass.ru ().

    Домашнее задание

    1. Определи порядок действий в данных выражениях. Найди значение выражений.

    2. Определи, в каком выражении такой порядок выполнения действий:

    1. умножение; 2. деление;. 3. сложение; 4. вычитание; 5. сложение. Найди значение данного выражения.

    3. Составь три выражения, в которых такой порядок выполнения действий:

    1. умножение; 2. сложение; 3. вычитание

    1. сложение; 2. вычитание; 3. сложение

    1. умножение; 2. деление; 3. сложение

    Найди значение этих выражений.

    Когда мы работаем с различными выражениями, включающими в себя цифры, буквы и переменные, нам приходится выполнять большое количество арифметических действий. Когда мы делаем преобразование или вычисляем значение, очень важно соблюдать правильную очередность этих действий. Иначе говоря, арифметические действия имеют свой особый порядок выполнения.

    Yandex.RTB R-A-339285-1

    В этой статье мы расскажем, какие действия надо делать в первую очередь, а какие после. Для начала разберем несколько простых выражений, в которых есть только переменные или числовые значения, а также знаки деления, умножения, вычитания и сложения. Потом возьмем примеры со скобками и рассмотрим, в каком порядке следует вычислять их. В третьей части мы приведем нужный порядок преобразований и вычислений в тех примерах, которые включают в себя знаки корней, степеней и других функций.

    Определение 1

    В случае выражений без скобок порядок действий определяется однозначно:

    1. Все действия выполняются слева направо.
    2. В первую очередь мы выполняем деление и умножение, во вторую – вычитание и сложение.

    Смысл этих правил легко уяснить. Традиционный порядок записи слева направо определяет основную последовательность вычислений, а необходимость сначала умножить или разделить объясняется самой сутью этих операций.

    Возьмем для наглядности несколько задач. Мы использовали только самые простые числовые выражения, чтобы все вычисления можно было провести в уме. Так можно быстрее запомнить нужный порядок и быстро проверить результаты.

    Пример 1

    Условие: вычислите, сколько будет 7 − 3 + 6 .

    Решение

    В нашем выражении скобок нет, умножение и деление также отсутствуют, поэтому выполняем все действия в указанном порядке. Сначала вычитаем три из семи, затем прибавляем к остатку шесть и в итоге получаем десять. Вот запись всего решения:

    7 − 3 + 6 = 4 + 6 = 10

    Ответ: 7 − 3 + 6 = 10 .

    Пример 2

    Условие: в каком порядке нужно выполнять вычисления в выражении 6: 2 · 8: 3 ?

    Решение

    Чтобы дать ответ на этот вопрос, перечитаем правило для выражений без скобок, сформулированное нами до этого. У нас здесь есть только умножение и деление, значит, мы сохраняем записанный порядок вычислений и считаем последовательно слева направо.

    Ответ: сначала выполняем деление шести на два, результат умножаем на восемь и получившееся в итоге число делим на три.

    Пример 3

    Условие: подсчитайте, сколько будет 17 − 5 · 6: 3 − 2 + 4: 2 .

    Решение

    Сначала определим верный порядок действий, поскольку у нас здесь есть все основные виды арифметических операций – сложение, вычитание, умножение, деление. Первым делом нам надо разделить и умножить. Эти действия не имеют приоритета друг перед другом, поэтому выполняем их в написанном порядке справа налево. То есть 5 надо умножить на 6 и получить 30 , потом 30 разделить на 3 и получить 10 . После этого делим 4 на 2 , это 2 . Подставим найденные значения в исходное выражение:

    17 − 5 · 6: 3 − 2 + 4: 2 = 17 − 10 − 2 + 2

    Здесь уже нет ни деления, ни умножения, поэтому делаем оставшиеся вычисления по порядку и получаем ответ:

    17 − 10 − 2 + 2 = 7 − 2 + 2 = 5 + 2 = 7

    Ответ: 17 − 5 · 6: 3 − 2 + 4: 2 = 7 .

    Пока порядок выполнения действий не заучен твердо, можно ставить над знаками арифметических действий цифры, означающие порядок вычисления. Например, для задачи выше мы могли бы записать так:

    Если у нас есть буквенные выражения, то с ними мы поступаем точно так же: сначала умножаем и делим, затем складываем и вычитаем.

    Что такое действия первой и второй ступени

    Иногда в справочниках все арифметические действия делят на действия первой и второй ступени. Сформулируем нужное определение.

    К действиям первой ступени относятся вычитание и сложение, второй – умножение и деление.

    Зная эти названия, мы можем записать данное ранее правило относительно порядка действий так:

    Определение 2

    В выражении, в котором нет скобок, сначала надо выполнить действия второй ступени в направлении слева направо, затем действия первой ступени (в том же направлении).

    Порядок вычислений в выражениях со скобками

    Скобки сами по себе являются знаком, который сообщает нам нужный порядок выполнения действий. В таком случае нужное правило можно записать так:

    Определение 3

    Если в выражении есть скобки, то первым делом выполняется действие в них, после чего мы умножаем и делим, а затем складываем и вычитаем по направлению слева направо.

    Что касается самого выражения в скобках, его можно рассматривать в качестве составной части основного выражения. При подсчете значения выражения в скобках мы сохраняем все тот же известный нам порядок действий. Проиллюстрируем нашу мысль примером.

    Пример 4

    Условие: вычислите, сколько будет 5 + (7 − 2 · 3) · (6 − 4) : 2 .

    Решение

    В данном выражении есть скобки, поэтому начнем с них. Первым делом вычислим, сколько будет 7 − 2 · 3 . Здесь нам надо умножить 2 на 3 и вычесть результат из 7:

    7 − 2 · 3 = 7 − 6 = 1

    Считаем результат во вторых скобках. Там у нас всего одно действие: 6 − 4 = 2 .

    Теперь нам нужно подставить получившиеся значения в первоначальное выражение:

    5 + (7 − 2 · 3) · (6 − 4) : 2 = 5 + 1 · 2: 2

    Начнем с умножения и деления, потом выполним вычитание и получим:

    5 + 1 · 2: 2 = 5 + 2: 2 = 5 + 1 = 6

    На этом вычисления можно закончить.

    Ответ: 5 + (7 − 2 · 3) · (6 − 4) : 2 = 6 .

    Не пугайтесь, если в условии у нас содержится выражение, в котором одни скобки заключают в себе другие. Нам надо только применять правило выше последовательно по отношению ко всем выражениям в скобках. Возьмем такую задачу.

    Пример 5

    Условие: вычислите, сколько будет 4 + (3 + 1 + 4 · (2 + 3)) .

    Решение

    У нас есть скобки в скобках. Начинаем с 3 + 1 + 4 · (2 + 3) , а именно с 2 + 3 . Это будет 5 . Значение надо будет подставить в выражение и подсчитать, что 3 + 1 + 4 · 5 . Мы помним, что сначала надо умножить, а потом сложить: 3 + 1 + 4 · 5 = 3 + 1 + 20 = 24 . Подставив найденные значения в исходное выражение, вычислим ответ: 4 + 24 = 28 .

    Ответ: 4 + (3 + 1 + 4 · (2 + 3)) = 28 .

    Иначе говоря, при вычислении значения выражения, включающего скобки в скобках, мы начинаем с внутренних скобок и продвигаемся к внешним.

    Допустим, нам надо найти, сколько будет (4 + (4 + (4 − 6: 2)) − 1) − 1 . Начинаем с выражения во внутренних скобках. Поскольку 4 − 6: 2 = 4 − 3 = 1 , исходное выражение можно записать как (4 + (4 + 1) − 1) − 1 . Снова обращаемся к внутренним скобкам: 4 + 1 = 5 . Мы пришли к выражению (4 + 5 − 1) − 1 . Считаем 4 + 5 − 1 = 8 и в итоге получаем разность 8 — 1 , результатом которой будет 7 .

    Порядок вычисления в выражениях со степенями, корнями, логарифмами и иными функциями

    Если у нас в условии стоит выражение со степенью, корнем, логарифмом или тригонометрической функцией (синусом, косинусом, тангенсом и котангенсом) или иными функциями, то первым делом мы вычисляем значение функции. После этого мы действуем по правилам, указанным в предыдущих пунктах. Иначе говоря, функции по степени важности приравниваются к выражению, заключенному в скобки.

    Разберем пример такого вычисления.

    Пример 6

    Условие: найдите, сколько будет (3 + 1) · 2 + 6 2: 3 − 7 .

    Решение

    У нас есть выражение со степенью, значение которого надо найти в первую очередь. Считаем: 6 2 = 36 . Теперь подставим результат в выражение, после чего оно примет вид (3 + 1) · 2 + 36: 3 − 7 .

    (3 + 1) · 2 + 36: 3 − 7 = 4 · 2 + 36: 3 − 7 = 8 + 12 − 7 = 13

    Ответ: (3 + 1) · 2 + 6 2: 3 − 7 = 13 .

    В отдельной статье, посвященной вычислению значений выражений, мы приводим и другие, более сложные примеры подсчетов в случае выражений с корнями, степенью и др. Рекомендуем вам с ней ознакомиться.

    Если вы заметили ошибку в тексте, пожалуйста, выделите её и нажмите Ctrl+Enter

    Начальная школа подходит к концу, скоро ребёнок шагнёт в углубленный мир математики. Но уже в этот период школьник сталкивается с трудностями науки. Выполняя простое задание, ребёнок путается, теряется, что в результате приводит к отрицательной отметке за выполненную работу. Чтобы избежать подобных неприятностей, нужно при решении примеров, уметь ориентироваться в порядке, по которому нужно решать пример. Не верно распределив действия, ребёнок не правильно выполняет задание. В статье раскрываются основные правила решения примеров, содержащих в себе весь спектр математических вычислений, включая скобки. Порядок действий в математике 4 класс правила и примеры.

    Перед выполнением задания попросите своё чадо пронумеровать действия, которые он собирается выполнить. Если возникли затруднения – помогите.

    Некоторые правила, которые необходимо соблюдать при решении примеров без скобок:

    Если в задании необходимо выполнить ряд действий, нужно сначала выполнить деление или умножение, затем . Все действия выполняются по ходу письма. В противном случае, результат решения будет не верным.

    Если в примере требуется выполнить , выполняем по порядку, слева направо.

    27-5+15=37 (при решении примера руководствуемся правилом. Сначала выполняем вычитание, затем – сложение).

    Научите ребёнка всегда планировать и нумеровать выполняемые действия.

    Ответы на каждое решённое действие записываются над примером. Так ребёнку гораздо легче будет ориентироваться в действиях.

    Рассмотрим ещё один вариант, где необходимо распределить действия по порядку:

    Как видим, при решении соблюдено правило, сначала ищем произведение, после — разность.

    Это простые примеры, при решении которых, необходима внимательность. Многие дети впадают в ступор при виде задания, в котором присутствует не только умножение и деление, но и скобки. У школьника, не знающего порядок выполнения действий, возникают вопросы, которые мешают выполнить задание.

    Как говорилось в правиле, сначала найдём произведение или частное, а потом всё остальное. Но тут же есть скобки! Как поступить в этом случае?

    Решение примеров со скобками

    Разберём конкретный пример:

    • При выполнении данного задания, сначала найдём значение выражения, заключённого в скобки.
    • Начать следует с умножения, далее – сложение.
    • После того, как выражение в скобках решено, приступаем к действиям вне их.
    • По правилам порядка действий, следующим шагом будет умножение.
    • Завершающим этапом станет .

    Как видим на наглядном примере, все действия пронумерованы. Для закрепления темы предложите ребёнку решить самостоятельно несколько примеров:

    Порядок, по которому следует вычислять значение выражения уже расставлен. Ребёнку останется только выполнить непосредственно решение.

    Усложним задачу. Пусть ребёнок найдёт значение выражений самостоятельно.

    7*3-5*4+(20-19) 14+2*3-(13-9)
    17+2*5+(28-2) 5*3+15-(2-1*2)
    24-3*2-(56-4*3) 14+12-3*(21-7)

    Приучите ребёнка решать все задания в черновом варианте. В таком случае, у школьника будет возможность исправить не верное решение или помарки. В рабочей тетради исправления не допустимы. Выполняя самостоятельно задания, дети видят свои ошибки.

    Родители, в свою очередь, должны обратить внимание на ошибки, помочь ребёнку разобраться и исправить их. Не стоит нагружать мозг школьника большими объёмами заданий. Такими действиями вы отобьёте стремление ребёнка к знаниям. Во всём должно быть чувство меры.

    Делайте перерыв. Ребёнок должен отвлекаться и отдыхать от занятий. Главное помнить, что не все обладают математическим складом ума. Может из вашего ребёнка вырастет знаменитый философ.

    Порядок решения примеров с умножением и делением. Учебно-методический материал по математике (3 класс) на тему: Примеры на порядок действий

    Числовые,буквенные выражения и выражения с переменными в своей записи могут содержать знаки различных арифметических действий. При преобразовании выражений и вычислении значений выражений действия выполняются в определенной очередности, иными словами, нужно соблюдать порядок выполнения действий .

    В этой статье мы разберемся, какие действия следует выполнять сначала, а какие следом за ними. Начнем с самых простых случаев, когда выражение содержит лишь числа или переменные, соединенные знаками плюс, минус, умножить и разделить. Дальше разъясним, какого порядка выполнения действий следует придерживаться в выражениях со скобками. Наконец, рассмотрим, в какой последовательности выполняются действия в выражениях, содержащих степени, корни и другие функции.

    Навигация по странице.

    В школе дается следующее правило, определяющее порядок выполнения действий в выражениях без скобок :

  • действия выполняются по порядку слева направо,
  • причем сначала выполняется умножение и деление, а затем – сложение и вычитание.
  • Озвученное правило воспринимается достаточно естественно. Выполнение действий по порядку слева направо объясняется тем, что у нас принято вести записи слева направо. А то, что умножение и деление выполняется перед сложением и вычитанием объясняется смыслом, который в себе несут эти действия.

    Рассмотрим несколько примеров применения этого правила. Для примеров будем брать простейшие числовые выражения, чтобы не отвлекаться на вычисления, а сосредоточиться именно на порядке выполнения действий.

    Выполните действия 7−3+6 .

    Исходное выражение не содержит скобок, а также оно не содержит умножения и деления. Поэтому нам следует выполнить все действия по порядку слева направо, то есть, сначала мы от 7 отнимаем 3 , получаем 4 , после чего к полученной разности 4 прибавляем 6 , получаем 10 .

    Кратко решение можно записать так: 7−3+6=4+6=10 .

    Укажите порядок выполнения действий в выражении 6:2·8:3 .

    Чтобы ответить на вопрос задачи, обратимся к правилу, указывающему порядок выполнения действий в выражениях без скобок. В исходном выражении содержатся лишь действия умножения и деления, а согласно правилу, их нужно выполнять по порядку слева направо.

    сначала 6 делим на 2 , это частное умножаем на 8 , наконец, полученный результат делим на 3.

    Вычислите значение выражения 17−5·6:3−2+4:2 .

    Сначала определим, в каком порядке следует выполнять действия в исходном выражении. Оно содержит и умножение с делением, и сложение с вычитанием. Сначала слева направо нужно выполнить умножение и деление. Так 5 умножаем на 6 , получаем 30 , это число делим на 3 , получаем 10 . Теперь 4 делим на 2 , получаем 2 . Подставляем в исходное выражение вместо 5·6:3 найденное значение 10 , а вместо 4:2 — значение 2 , имеем 17−5·6:3−2+4:2=17−10−2+2 .

    В полученном выражении уже нет умножения и деления, поэтому остается по порядку слева направо выполнить оставшиеся действия: 17−10−2+2=7−2+2=5+2=7 .

    На первых порах, чтобы не перепутать порядок выполнения действий при вычислении значения выражения, удобно над знаками действий расставить цифры, соответствующие порядку их выполнения. Для предыдущего примера это выглядело бы так: .

    Этого же порядка выполнения действий – сначала умножение и деление, затем сложение и вычитание — следует придерживаться и при работе с буквенными выражениями.

    Действия первой и второй ступени

    В некоторых учебниках по математике встречается разделение арифметических действий на действия первой и второй ступени. Разберемся с этим.

    Действиями первой ступени называют сложение и вычитание, а умножение и деление называют действиями второй ступени .

    В этих терминах правило из предыдущего пункта, определяющее порядок выполнения действий, запишется так: если выражение не содержит скобок, то по порядку слева направо сначала выполняются действия второй ступени (умножение и деление), затем – действия первой ступени (сложение и вычитание).

    Порядок выполнения арифметических действий в выражениях со скобками

    Выражения часто содержат скобки, указывающие порядок выполнения действий. В этом случае правило, задающее порядок выполнения действий в выражениях со скобками , формулируется так: сначала выполняются действия в скобках, при этом также по порядку слева направо выполняется умножение и деление, затем – сложение и вычитание.

    Итак, выражения в скобках рассматриваются как составные части исходного выражения, и в них сохраняется уже известный нам порядок выполнения действий. Рассмотрим решения примеров для большей ясности.

    Выполните указанные действия 5+(7−2·3)·(6−4):2 .

    Выражение содержит скобки, поэтому сначала выполним действия в выражениях, заключенных в эти скобки. Начнем с выражения 7−2·3 . В нем нужно сначала выполнить умножение, и только потом вычитание, имеем 7−2·3=7−6=1 . Переходим ко второму выражению в скобках 6−4 . Здесь лишь одно действие – вычитание, выполняем его 6−4=2 .

    Подставляем полученные значения в исходное выражение: 5+(7−2·3)·(6−4):2=5+1·2:2 . В полученном выражении сначала выполняем слева направо умножение и деление, затем – вычитание, получаем 5+1·2:2=5+2:2=5+1=6 . На этом все действия выполнены, мы придерживались такого порядка их выполнения: 5+(7−2·3)·(6−4):2 .

    Запишем краткое решение: 5+(7−2·3)·(6−4):2=5+1·2:2=5+1=6 .

    Бывает, что выражение содержит скобки в скобках. Этого бояться не стоит, нужно лишь последовательно применять озвученное правило выполнения действий в выражениях со скобками. Покажем решение примера.

    Выполните действия в выражении 4+(3+1+4·(2+3)) .

    Это выражение со скобками, это означает, что выполнение действий нужно начинать с выражения в скобках, то есть, с 3+1+4·(2+3) . Это выражение также содержит скобки, поэтому нужно сначала выполнить действия в них. Сделаем это: 2+3=5 . Подставив найденное значение, получаем 3+1+4·5 . В этом выражении сначала выполняем умножение, затем – сложение, имеем 3+1+4·5=3+1+20=24 . Исходное значение, после подстановки этого значения, принимает вид 4+24 , и остается лишь закончить выполнение действий: 4+24=28 .

    Вообще, когда в выражении присутствуют скобки в скобках, то часто бывает удобно выполнение действий начинать с внутренних скобок и продвигаться к внешним.

    Например, пусть нам нужно выполнить действия в выражении (4+(4+(4−6:2))−1)−1 . Сначала выполняем действия во внутренних скобках, так как 4−6:2=4−3=1 , то после этого исходное выражение примет вид (4+(4+1)−1)−1 . Опять выполняем действие во внутренних скобках, так как 4+1=5 , то приходим к следующему выражению (4+5−1)−1 . Опять выполняем действия в скобках: 4+5−1=8 , при этом приходим к разности 8−1 , которая равна 7 .

    Порядок выполнения действий в выражениях с корнями, степенями, логарифмами и другими функциями

    Если в выражение входят степени, корни, логарифмы, синус, косинус, тангенс и котангенс, а также другие функции, то их значения вычисляются до выполнения остальных действий, при этом также учитываются правила из предыдущих пунктов, задающие порядок выполнения действий. Иными словами, перечисленные вещи, грубо говоря, можно считать заключенными в скобки, а мы знаем, что сначала выполняются действия в скобках.

    Рассмотрим решения примеров.

    Выполните действия в выражении (3+1)·2+6 2:3−7 .

    В этом выражении содержится степень 6 2 , ее значение нужно вычислить до выполнения остальных действий. Итак, выполняем возведение в степень: 6 2 =36 . Подставляем это значение в исходное выражение, оно примет вид (3+1)·2+36:3−7 .

    Дальше все понятно: выполняем действия в скобках, после чего остается выражение без скобок, в котором по порядку слева направо сначала выполняем умножение и деление, а затем – сложение и вычитание. Имеем (3+1)·2+36:3−7=4·2+36:3−7= 8+12−7=13 .

    Другие, в том числе и более сложные примеры выполнения действий в выражениях с корнями, степенями и т.п., Вы можете посмотреть в статье вычисление значений выражений.

    cleverstudents.ru

    Онлайн игры,тренажеры,презентации,уроки,энциклопедии,статьи

    Post navigation

    Примеры со скобками, урок с тренажерами.

    Мы рассмотрим в этой статье три варианта примеров:

    1. Примеры со скобками (действия сложения и вычитания)

    2. Примеры со скобками (сложение, вычитание, умножение, деление)

    3. Примеры, в которых много действий

    1 Примеры со скобками (действия сложения и вычитания)

    Рассмотрим три примера. В каждом из них порядок действий обозначен цифрами красного цвета:

    Мы видим, что порядок действий в каждом примере будет разный, хотя числа и знаки одинаковые. Это происходит потому, что во втором и третьем примере есть скобки.

  • Если в примере нет скобок , мы выполняем все действия по порядку, слева направо.
  • Если в примере есть скобки , то сначала мы выполняем действия в скобках, и лишь потом все остальные действия, начиная слева направо.
  • *Это правило для примеров без умножения и деления. Правила для примеров со скобками, включающих действия умножения и деления мы рассмотрим во второй части этой статьи.

    Чтобы не запутаться в примере со скобками, можно превратить его в обычный пример, без скобок. Для этого результат, полученный в скобках, записываем над скобками, далее переписываем весь пример, записывая вместо скобок этот результат, и далее выполняем все действия по порядку, слева направо:

    В несложных примерах можно все эти операции производить в уме. Главное — сначала выполнить действие в скобках и запомнить результат, а затем считать по порядку, слева направо.

    А теперь — тренажеры!

    1) Примеры со скобками в пределах до 20. Онлайн тренажер.

    2) Примеры со скобками в пределах до 100. Онлайн тренажер.

    3) Примеры со скобками. Тренажер №2

    4) Вставь пропущенное число — примеры со скобками. Тренажер

    2 Примеры со скобками (сложение, вычитание, умножение, деление)

    Теперь рассмотрим примеры, в которых кроме сложения и вычитания есть умножение и деление.

    Сначала рассмотрим примеры без скобок:

  • Если в примере нет скобок , сначала выполняем действия умножения и деления по порядку, слева направо. Затем — действия сложения и вычитания по порядку, слева направо.
  • Если в примере есть скобки , то сначала мы выполняем действия в скобках, затем умножение и деление, и затем — сложение и вычитание начиная слева направо.
  • Есть одна хитрость, как не запутаться при решении примеров на порядок действий. Если нет скобок, то выполняем действия умножения и деления, далее переписываем пример, записывая вместо этих действий полученные результаты. Затем выполняем сложение и вычитание по порядку:

    Если в примере есть скобки, то сначала нужно избавиться от скобок: переписать пример, записывая вместо скобок полученный в них результат. Затем нужно выделить мысленно части примера, разделенные знаками «+» и «-«, и посчитать каждую часть отдельно. Затем выполнить сложение и вычитание по порядку:

    3 Примеры, в которых много действий

    Если в примере много действий, то удобнее будет не расставлять порядок действий во всем примере, а выделить блоки, и решить каждый блок отдельно. Для этого находим свободные знаки «+» и «–» (свободные — значит не в скобках, на рисунке показаны стрелочками).

    Эти знаки и будут делить наш пример на блоки:

    Выполняя действия в каждом блоке не забываем про порядок действий, приведенный выше в статье. Решив каждый блок, выполняем действия сложения и вычитания по порядку.

    А теперь закрепляем решение примеров на порядок действий на тренажерах!

    1. Примеры со скобками в пределах чисел до 100, действия сложения, вычитания, умножения и деления. Онлайн тренажер.

    2. Тренажер по математике 2 — 3 класс «Расставь порядок действий (буквенные выражения).»

    3. Порядок действий (расставляем порядок и решаем примеры)

    Порядок действий в математике 4 класс

    Начальная школа подходит к концу, скоро ребёнок шагнёт в углубленный мир математики. Но уже в этот период школьник сталкивается с трудностями науки. Выполняя простое задание, ребёнок путается, теряется, что в результате приводит к отрицательной отметке за выполненную работу. Чтобы избежать подобных неприятностей, нужно при решении примеров, уметь ориентироваться в порядке, по которому нужно решать пример. Не верно распределив действия, ребёнок не правильно выполняет задание. В статье раскрываются основные правила решения примеров, содержащих в себе весь спектр математических вычислений, включая скобки. Порядок действий в математике 4 класс правила и примеры.

    Перед выполнением задания попросите своё чадо пронумеровать действия, которые он собирается выполнить. Если возникли затруднения – помогите.

    Некоторые правила, которые необходимо соблюдать при решении примеров без скобок:

    Если в задании необходимо выполнить ряд действий, нужно сначала выполнить деление или умножение, затем сложение. Все действия выполняются по ходу письма. В противном случае, результат решения будет не верным.

    Если в примере требуется выполнить сложение и вычитание, выполняем по порядку, слева направо.

    27-5+15=37 (при решении примера руководствуемся правилом. Сначала выполняем вычитание, затем – сложение).

    Научите ребёнка всегда планировать и нумеровать выполняемые действия.

    Ответы на каждое решённое действие записываются над примером. Так ребёнку гораздо легче будет ориентироваться в действиях.

    Рассмотрим ещё один вариант, где необходимо распределить действия по порядку:

    Как видим, при решении соблюдено правило, сначала ищем произведение, после — разность.

    Это простые примеры, при решении которых, необходима внимательность. Многие дети впадают в ступор при виде задания, в котором присутствует не только умножение и деление, но и скобки. У школьника, не знающего порядок выполнения действий, возникают вопросы, которые мешают выполнить задание.

    Как говорилось в правиле, сначала найдём произведение или частное, а потом всё остальное. Но тут же есть скобки! Как поступить в этом случае?

    Решение примеров со скобками

    Разберём конкретный пример:

  • При выполнении данного задания, сначала найдём значение выражения, заключённого в скобки.
  • Начать следует с умножения, далее – сложение.
  • После того, как выражение в скобках решено, приступаем к действиям вне их.
  • По правилам порядка действий, следующим шагом будет умножение.
  • Завершающим этапом станет вычитание.
  • Как видим на наглядном примере, все действия пронумерованы. Для закрепления темы предложите ребёнку решить самостоятельно несколько примеров:

    Порядок, по которому следует вычислять значение выражения уже расставлен. Ребёнку останется только выполнить непосредственно решение.

    Усложним задачу. Пусть ребёнок найдёт значение выражений самостоятельно.

    7*3-5*4+(20-19) 14+2*3-(13-9)
    17+2*5+(28-2) 5*3+15-(2-1*2)
    24-3*2-(56-4*3) 14+12-3*(21-7)

    Приучите ребёнка решать все задания в черновом варианте. В таком случае, у школьника будет возможность исправить не верное решение или помарки. В рабочей тетради исправления не допустимы. Выполняя самостоятельно задания, дети видят свои ошибки.

    Родители, в свою очередь, должны обратить внимание на ошибки, помочь ребёнку разобраться и исправить их. Не стоит нагружать мозг школьника большими объёмами заданий. Такими действиями вы отобьёте стремление ребёнка к знаниям. Во всём должно быть чувство меры.

    Делайте перерыв. Ребёнок должен отвлекаться и отдыхать от занятий. Главное помнить, что не все обладают математическим складом ума. Может из вашего ребёнка вырастет знаменитый философ.

    detskoerazvitie.info

    Урок по математике 2 класс Порядок действий в выражениях со скобками.

    Успейте воспользоваться скидками до 50% на курсы «Инфоурок»

    Цель: 1.

    2.

    3. Закрепить знание таблицы умножения и деления на 2 – 6, понятия делителя и

    4. Учить работать в парах с целью развития коммуникативных качеств.

    Оборудование * : + — (), геометрический материал.

    Раз, два – выше голова.

    Три, четыре – руки шире.

    Пять, шесть – всем присесть.

    Семь, восемь – лень отбросим.

    Но сначала придется узнать его название. Для этого нужно выполнить несколько заданий:

    6 + 6 + 6 … 6 * 4 6 * 4 + 6… 6 * 5 – 6 14 дм 5 см… 4 дм 5 см

    Пока мы вспоминали о порядке действий в выражениях, с замком происходили чудеса. Мы были только что у ворот, а теперь попали в коридор. Смотрите, дверь. А на ней замок. Откроем?

    1. Из числа 20 вычесть частное чисел 8 и 2.

    2. Разность чисел 20 и 8 разделить на 2.

    — Чем отличаются результаты?

    — Кто сможет назвать тему нашего урока?

    (на массажных ковриках)

    По дорожке, по дорожке

    Скачем мы на правой ножке,

    Скачем мы на левой ножке.

    По тропинке побежим,

    Наше предположение было полностью правильно7

    Где выполняются действия сначала, если в выражении есть скобки?

    Смотрите перед нами «живые примеры». Давайте «оживим» их.

    * : + — ().

    m – c * (a + d) + x

    k: b + (a – c) * t

    6. Работа в парах.

    Для их решения вам понадобиться геометрический материал.

    Учащиеся выполняют задания в парах. После выполнения проверка работы пар у доски.

    Что нового вы узнали?

    8. Домашнее задание.

    Тема: Порядок действий в выражениях со скобками.

    Цель: 1. Вывести правило порядка действий в выражениях со скобками, содержащих все

    4 арифметических действия,

    2. Формировать способность к практическому применению правила,

    4.Учить работать в парах с целью развития коммуникативных качеств.

    Оборудование : учебник, тетради, карточки со знаками действий * : + — (), геометрический материал.

    1 .Физминутка.

    Девять, десять – тихо сесть.

    2. Актуализация опорных знаний.

    Сегодня мы с вами отправляемся в очередное путешествие по стране Знаний городу математика. Нам предстоит посетить один дворец. Что-то я забыла его название. Но не будем расстраиваться, вы сами сможете мне подсказать его название. Пока я переживала, мы подошли к воротам дворца. Войдем?

    1. Сравните выражения:

    2. Расшифруй слово.

    3. Постановка проблемы. Открытие нового.

    Так как же называется дворец?

    А когда в математике мы говорим о порядке?

    Что вы уже знаете о порядке выполнения действий в выражениях?

    — Интересно, нам предлагают записать и решить выражения (учитель читает выражения, учащиеся записывают их и решают).

    20 – 8: 2

    (20 – 8) : 2

    Молодцы. А что интересного в этих выражениях?

    Посмотрите на выражения и их результаты.

    — Что общего в записи выражений?

    — Как вы думаете, почему получились разные результаты, ведь числа были одинаковые?

    Кто рискнет сформулировать правило выполнения действий в выражениях со скобками?

    Правильность этого ответа мы сможем проверить в другой комнате. Отправляемся туда.

    4. Физминутка.

    И по этой же дорожке

    До горы мы добежим.

    Стоп. Немножко отдохнем

    И опять пешком пойдем.

    5. Первичное закрепление изученного.

    Вот мы и пришли.

    Нам нужно решить еще два выражения, чтобы проверить правильность нашего предположения.

    6 * (33 – 25) 54: (6 + 3) 25 – 5 * (9 – 5) : 2

    Для проверки правильности предположения откроем учебники на стр. 33 и прочитаем правило.

    Как нужно выполнять действия после решения в скобках?

    На доске написаны буквенные выражения и лежат карточки со знаками действий * : + — (). Дети выходят к доске по одному, берут карточку с тем действием, которое нужно сделать сначала, потом выходит второй ученик и берет карточку со вторым действием и т. д.

    а + (а –в)

    а * (в +с) : d t

    m c * ( a + d ) + x

    k : b + ( a c ) * t

    (a – b) : t + d

    6. Работа в парах.

    Знание порядка действий необходимо не только для решения примеров, но и при решении задач мы тоже сталкиваемся с этим правилом. Сейчас вы в этом убедитесь работая в парах. Вам нужно будет решить задачи из № 3 стр. 33.

    7. Итог.

    По какому дворцу мы с вами сегодня путешествовали?

    Вам понравился урок?

    Как нужно выполнять действия в выражениях со скобками?

    • Можно ли оформить договор купли-продажи квартиры, купленной за материнский капитал? В настоящей момент каждой семье, в которой родился или которая усыновила второго ребенка, государство предоставляет возможность […]
    • Особенности бухгалтерского учета субсидий Государство стремится поддержать малое и среднее предпринимательство. Такая поддержка наиболее часто выражается в форме предоставления субсидий – безвозмездных выплат из […]
    • Работа вахтой в Москве — свежие вакансии прямых работодателей логистические компании; склады; Дополнительный плюс работы вахтовым методом заключается в том, что работник получает от компании проживание (в […]
    • Ходатайство об уменьшении размера исковых требований Один из видов уточнения иска — ходатайство об уменьшении размера исковых требований. Когда истец неправильно определил цену иска. Или ответчик частично исполнил […]
    • Как правильно париться в бане Банная процедура с парением — это целая наука. Основные правила парильщика: не торопиться, наибольшее удовольствие от бани — когда можно не спеша несколько раз зайти в парилку с […]
    • Школьная Энциклопедия Nav view search Login Form Законы Кеплера о движении планет Подробности Категория: Этапы развития астрономии Опубликовано 20.09.2012 13:44 Просмотров: 25396 «Он жил в эпоху, когда ещё не […]

    4 класс. Математика. Выражение и его значение. Порядок выполнения действий — Выражение и его значение. Порядок выполнения действий

    Комментарии преподавателя

    На дан­ном уроке мы рас­смот­рим вы­ра­же­ние и его зна­че­ние, а также по­ря­док вы­пол­не­ния дей­ствий. Для на­ча­ла вспом­ним, что на­зы­ва­ют чис­ло­вым вы­ра­же­ни­ем.

    Чис­ло­вое вы­ра­же­ние – за­пись, со­сто­я­щая из чисел, со­еди­нен­ных ариф­ме­ти­че­ски­ми дей­стви­я­ми.

    Вы­бе­ри­те чис­ло­вые вы­ра­же­ния

    1.

    2.

    3.

    4.

    5.

    Вто­рая за­пись на­зы­ва­ет­ся ра­вен­ство, по­это­му она лиш­няя. Осталь­ные за­пи­си на­зы­ва­ют­ся чис­ло­вы­ми вы­ра­же­ни­я­ми. Если вы­пол­нить ука­зан­ные дей­ствия в этих чис­ло­вых вы­ра­же­ни­ях, то най­дем зна­че­ния вы­ра­же­ний.

    Мы знаем че­ты­ре ариф­ме­ти­че­ских дей­ствия: сло­же­ние, вы­чи­та­ние, умно­же­ние, де­ле­ние. В одном вы­ра­же­нии можно вы­пол­нять несколь­ко дей­ствий. Чтобы найти зна­че­ние та­ко­го вы­ра­же­ния, нужно вы­пол­нять дей­ствия сле­ду­ю­щим об­ра­зом:

    Пра­ви­ло 1

    Если чис­ло­вое вы­ра­же­ние со­дер­жит толь­ко дей­ствия сло­же­ния и вы­чи­та­ния, то дей­ствия вы­пол­ня­ют по по­ряд­ку слева на­пра­во.

    Если чис­ло­вое вы­ра­же­ние со­дер­жит толь­ко дей­ствия умно­же­ния и де­ле­ния, то дей­ствия вы­пол­ня­ют также по по­ряд­ку слева на­пра­во.

    Рас­ставь­те по­ря­док дей­ствий и вы­пол­ни­те вы­чис­ле­ния:

    1. 83 + 12 – 25 + 20

    2. 49 : 7 ∙ 4 : 28

    Ре­ше­ние:

    83 + 12 – 25 + 20 = 90 (по­ря­док слева на­пра­во, так как толь­ко дей­ствия сло­же­ние и вы­чи­та­ние)

    49:7 ∙ 4 : 28 = 1 (по­ря­док слева на­пра­во, так как толь­ко дей­ствия умно­же­ние и де­ле­ние)

    Ответ: 1. 90; 2. 1

    Пра­ви­ло 2

    Если чис­ло­вое вы­ра­же­ние со­дер­жит не толь­ко сло­же­ние и вы­чи­та­ние, но и умно­же­ние с де­ле­ни­ем, то сна­ча­ла вы­пол­ня­ют умно­же­ние и де­ле­ние по по­ряд­ку слева на­пра­во, а потом сло­же­ние и вы­чи­та­ние слева на­пра­во.

    За­да­ние

    Рас­ставь­те по­ря­док дей­ствий и вы­пол­ни­те вы­чис­ле­ния:

    1. 114 – 9 ∙ 4 : 6

    2. 42 – 45 : 5 + 2 ∙ 7

    Ре­ше­ние:

    Ответ: 1. 108; 2. 47

    Пра­ви­ло 3

    Ино­гда за­пись вы­ра­же­ния со­дер­жит одну или несколь­ко пар ско­бок. В этом слу­чае сна­ча­ла на­хо­дят зна­че­ния вы­ра­же­ний в скоб­ках, а затем вы­пол­ня­ют дей­ствия по из­вест­ным нам пра­ви­лам.

    За­да­ние

    Рас­ставь­те по­ря­док дей­ствий и вы­пол­ни­те вы­чис­ле­ния:

    1. 480 : (30 – 24) ∙ 7

    2. 150 – (47 + 27 : 9)

    3. (340 – 280) : (27 : 9)

    Ре­ше­ние:

    Ответ: 1. 560; 2. 100; 3. 20

    На этом уроке мы вы­учи­ли пра­ви­ла по­ряд­ка вы­пол­не­ния дей­ствий при на­хож­де­нии зна­че­ния чис­ло­вых вы­ра­же­ний, а также под­кре­пи­ли эти зна­ния неко­то­ры­ми при­ме­ра­ми.


    источник конспекта — http://interneturok.ru/ru/school/matematika/4-klass/undefined/vyrazhenie-i-ego-znachenie-poryadok-vypolneniya-deystviy

    источник видео — http://www.youtube.com/watch?v=K80DkDbGW40

    источник презентации — http://ppt4web.ru/nachalnaja-shkola/porjadok-vypolnenija-dejjstvijj.html

    Порядок решения сложных примеров. Порядок выполнения действий — Гипермаркет знаний

    И вычислении значений выражений действия выполняются в определенной очередности, иными словами, нужно соблюдать порядок выполнения действий .

    В этой статье мы разберемся, какие действия следует выполнять сначала, а какие следом за ними. Начнем с самых простых случаев, когда выражение содержит лишь числа или переменные, соединенные знаками плюс, минус, умножить и разделить. Дальше разъясним, какого порядка выполнения действий следует придерживаться в выражениях со скобками. Наконец, рассмотрим, в какой последовательности выполняются действия в выражениях, содержащих степени, корни и другие функции.

    Навигация по странице.

    Сначала умножение и деление, затем сложение и вычитание

    В школе дается следующее правило, определяющее порядок выполнения действий в выражениях без скобок :

    • действия выполняются по порядку слева направо,
    • причем сначала выполняется умножение и деление, а затем – сложение и вычитание.

    Озвученное правило воспринимается достаточно естественно. Выполнение действий по порядку слева направо объясняется тем, что у нас принято вести записи слева направо. А то, что умножение и деление выполняется перед сложением и вычитанием объясняется смыслом, который в себе несут эти действия.

    Рассмотрим несколько примеров применения этого правила. Для примеров будем брать простейшие числовые выражения, чтобы не отвлекаться на вычисления, а сосредоточиться именно на порядке выполнения действий.

    Пример.

    Выполните действия 7−3+6 .

    Решение.

    Исходное выражение не содержит скобок, а также оно не содержит умножения и деления. Поэтому нам следует выполнить все действия по порядку слева направо, то есть, сначала мы от 7 отнимаем 3 , получаем 4 , после чего к полученной разности 4 прибавляем 6 , получаем 10 .

    Кратко решение можно записать так: 7−3+6=4+6=10 .

    Ответ:

    7−3+6=10 .

    Пример.

    Укажите порядок выполнения действий в выражении 6:2·8:3 .

    Решение.

    Чтобы ответить на вопрос задачи, обратимся к правилу, указывающему порядок выполнения действий в выражениях без скобок. В исходном выражении содержатся лишь действия умножения и деления, а согласно правилу, их нужно выполнять по порядку слева направо.

    Ответ:

    Сначала 6 делим на 2 , это частное умножаем на 8 , наконец, полученный результат делим на 3.

    Пример.

    Вычислите значение выражения 17−5·6:3−2+4:2 .

    Решение.

    Сначала определим, в каком порядке следует выполнять действия в исходном выражении. Оно содержит и умножение с делением, и сложение с вычитанием. Сначала слева направо нужно выполнить умножение и деление. Так 5 умножаем на 6 , получаем 30 , это число делим на 3 , получаем 10 . Теперь 4 делим на 2 , получаем 2 . Подставляем в исходное выражение вместо 5·6:3 найденное значение 10 , а вместо 4:2 — значение 2 , имеем 17−5·6:3−2+4:2=17−10−2+2 .

    В полученном выражении уже нет умножения и деления, поэтому остается по порядку слева направо выполнить оставшиеся действия: 17−10−2+2=7−2+2=5+2=7 .

    Ответ:

    17−5·6:3−2+4:2=7 .

    На первых порах, чтобы не перепутать порядок выполнения действий при вычислении значения выражения, удобно над знаками действий расставить цифры, соответствующие порядку их выполнения. Для предыдущего примера это выглядело бы так: .

    Этого же порядка выполнения действий – сначала умножение и деление, затем сложение и вычитание — следует придерживаться и при работе с буквенными выражениями.

    Действия первой и второй ступени

    В некоторых учебниках по математике встречается разделение арифметических действий на действия первой и второй ступени. Разберемся с этим.

    Определение.

    Действиями первой ступени называют сложение и вычитание, а умножение и деление называют действиями второй ступени .

    В этих терминах правило из предыдущего пункта, определяющее порядок выполнения действий, запишется так: если выражение не содержит скобок, то по порядку слева направо сначала выполняются действия второй ступени (умножение и деление), затем – действия первой ступени (сложение и вычитание).

    Порядок выполнения арифметических действий в выражениях со скобками

    Выражения часто содержат скобки, указывающие порядок выполнения действий . В этом случае правило, задающее порядок выполнения действий в выражениях со скобками , формулируется так: сначала выполняются действия в скобках, при этом также по порядку слева направо выполняется умножение и деление, затем – сложение и вычитание.

    Итак, выражения в скобках рассматриваются как составные части исходного выражения, и в них сохраняется уже известный нам порядок выполнения действий. Рассмотрим решения примеров для большей ясности.

    Пример.

    Выполните указанные действия 5+(7−2·3)·(6−4):2 .

    Решение.

    Выражение содержит скобки, поэтому сначала выполним действия в выражениях, заключенных в эти скобки. Начнем с выражения 7−2·3 . В нем нужно сначала выполнить умножение, и только потом вычитание, имеем 7−2·3=7−6=1 . Переходим ко второму выражению в скобках 6−4 . Здесь лишь одно действие – вычитание, выполняем его 6−4=2 .

    Подставляем полученные значения в исходное выражение: 5+(7−2·3)·(6−4):2=5+1·2:2 . В полученном выражении сначала выполняем слева направо умножение и деление, затем – вычитание, получаем 5+1·2:2=5+2:2=5+1=6 . На этом все действия выполнены, мы придерживались такого порядка их выполнения: 5+(7−2·3)·(6−4):2 .

    Запишем краткое решение: 5+(7−2·3)·(6−4):2=5+1·2:2=5+1=6 .

    Ответ:

    5+(7−2·3)·(6−4):2=6 .

    Бывает, что выражение содержит скобки в скобках. Этого бояться не стоит, нужно лишь последовательно применять озвученное правило выполнения действий в выражениях со скобками. Покажем решение примера.

    Пример.

    Выполните действия в выражении 4+(3+1+4·(2+3)) .

    Решение.

    Это выражение со скобками, это означает, что выполнение действий нужно начинать с выражения в скобках, то есть, с 3+1+4·(2+3) . Это выражение также содержит скобки, поэтому нужно сначала выполнить действия в них. Сделаем это: 2+3=5 . Подставив найденное значение, получаем 3+1+4·5 . В этом выражении сначала выполняем умножение, затем – сложение, имеем 3+1+4·5=3+1+20=24 . Исходное значение, после подстановки этого значения, принимает вид 4+24 , и остается лишь закончить выполнение действий: 4+24=28 .

    Ответ:

    4+(3+1+4·(2+3))=28 .

    Вообще, когда в выражении присутствуют скобки в скобках, то часто бывает удобно выполнение действий начинать с внутренних скобок и продвигаться к внешним.

    Например, пусть нам нужно выполнить действия в выражении (4+(4+(4−6:2))−1)−1 . Сначала выполняем действия во внутренних скобках, так как 4−6:2=4−3=1 , то после этого исходное выражение примет вид (4+(4+1)−1)−1 . Опять выполняем действие во внутренних скобках, так как 4+1=5 , то приходим к следующему выражению (4+5−1)−1 . Опять выполняем действия в скобках: 4+5−1=8 , при этом приходим к разности 8−1 , которая равна 7 .

    На данном уроке подробно рассмотрен порядок выполнения арифметических действий в выражениях без скобок и со скобками. Учащимся предоставляется возможность в ходе выполнения заданий определить, зависит ли значение выражений от порядка выполнения арифметических действий, узнать отличается ли порядок арифметических действий в выражениях без скобок и со скобками, потренироваться в применении изученного правила, найти и исправить ошибки, допущенные при определении порядка действий.

    В жизни мы постоянно выполняем какие-либо действия: гуляем, учимся, читаем, пишем, считаем, улыбаемся, ссоримся и миримся. Эти действия мы выполняем в разном порядке. Иногда их можно поменять местами, а иногда нет. Например, собираясь утром в школу, можно сначала сделать зарядку, затем заправить постель, а можно наоборот. Но нельзя сначала уйти в школу, а потом надеть одежду.

    А в математике обязательно ли выполнять арифметические действия в определенном порядке?

    Давайте проверим

    Сравним выражения:
    8-3+4 и 8-3+4

    Видим, что оба выражения совершенно одинаковы.

    Выполним действия в одном выражения слева направо, а в другом справа налево. Числами можно проставить порядок выполнения действий (рис. 1).

    Рис. 1. Порядок действий

    В первом выражении мы сначала выполним действие вычитания, а затем к результату прибавим число 4.

    Во втором выражении сначала найдем значение суммы, а потом из 8 вычтем полученный результат 7.

    Видим, что значения выражений получаются разные.

    Сделаем вывод: порядок выполнения арифметических действий менять нельзя .

    Узнаем правило выполнения арифметических действий в выражениях без скобок.

    Если в выражение без скобок входят только сложение и вычитание или только умножение и деление, то действия выполняют в том порядке, в каком они написаны.

    Потренируемся.

    Рассмотрим выражение

    В этом выражении имеются только действия сложения и вычитания. Эти действия называют действиями первой ступени .

    Выполняем действия слева направо по порядку (рис. 2).

    Рис. 2. Порядок действий

    Рассмотрим второе выражение

    В этом выражении имеются только действия умножения и деления — это действия второй ступени.

    Выполняем действия слева направо по порядку (рис. 3).

    Рис. 3. Порядок действий

    В каком порядке выполняются арифметические действия, если в выражении имеются не только действия сложения и вычитания, но и умножения и деления?

    Если в выражение без скобок входят не только действия сложения и вычитания, но и умножения и деления, или оба этих действия, то сначала выполняют по порядку (слева направо) умножение и деление, а затем сложение и вычитание.

    Рассмотрим выражение.

    Рассуждаем так. В этом выражении имеются действия сложения и вычитания, умножения и деления. Действуем по правилу. Сначала выполняем по порядку (слева направо) умножение и деление, а затем сложение и вычитание. Расставим порядок действий.

    Вычислим значение выражения.

    18:2-2*3+12:3=9-6+4=3+4=7

    В каком порядке выполняются арифметические действия, если в выражении имеются скобки?

    Если в выражении имеются скобки, то сначала вычисляют значение выражений в скобках.

    Рассмотрим выражение.

    30 + 6 * (13 — 9)

    Мы видим, что в этом выражении имеется действие в скобках, значит, это действие выполним первым, затем по порядку умножение и сложение. Расставим порядок действий.

    30 + 6 * (13 — 9)

    Вычислим значение выражения.

    30+6*(13-9)=30+6*4=30+24=54

    Как нужно рассуждать, чтобы правильно установить порядок арифметических действий в числовом выражении?

    Прежде чем приступить к вычислениям, надо рассмотреть выражение (выяснить, есть ли в нём скобки, какие действия в нём имеются) и только после этого выполнять действия в следующем порядке:

    1. действия, записанные в скобках;

    2. умножение и деление;

    3. сложение и вычитание.

    Схема поможет запомнить это несложное правило (рис. 4).

    Рис. 4. Порядок действий

    Потренируемся.

    Рассмотрим выражения, установим порядок действий и выполним вычисления.

    43 — (20 — 7) +15

    32 + 9 * (19 — 16)

    Будем действовать по правилу. В выражении 43 — (20 — 7) +15 имеются действия в скобках, а также действия сложения и вычитания. Установим порядок действий. Первым действием выполним действие в скобках, а затем по порядку слева направо вычитание и сложение.

    43 — (20 — 7) +15 =43 — 13 +15 = 30 + 15 = 45

    В выражении 32 + 9 * (19 — 16) имеются действия в скобках, а также действия умножения и сложения. По правилу первым выполним действие в скобках, затем умножение (число 9 умножаем на результат, полученный при вычитании) и сложение.

    32 + 9 * (19 — 16) =32 + 9 * 3 = 32 + 27 = 59

    В выражении 2*9-18:3 отсутствуют скобки, зато имеются действия умножения, деления и вычитания. Действуем по правилу. Сначала выполним слева направо умножение и деление, а затем от результата, полученного при умножении, вычтем результат, полученный при делении. То есть первое действие — умножение, второе — деление, третье — вычитание.

    2*9-18:3=18-6=12

    Узнаем, правильно ли определен порядок действий в следующих выражениях.

    37 + 9 — 6: 2 * 3 =

    18: (11 — 5) + 47=

    7 * 3 — (16 + 4)=

    Рассуждаем так.

    37 + 9 — 6: 2 * 3 =

    В этом выражении скобки отсутствуют, значит, сначала выполняем слева направо умножение или деление, затем сложение или вычитание. В данном выражении первое действие — деление, второе — умножение. Третье действие должно быть сложение, четвертое — вычитание. Вывод: порядок действий определен верно.

    Найдем значение данного выражения.

    37+9-6:2*3 =37+9-3*3=37+9-9=46-9=37

    Продолжаем рассуждать.

    Во втором выражении имеются скобки, значит, сначала выполняем действие в скобках, затем слева направо умножение или деление, сложение или вычитание. Проверяем: первое действие — в скобках, второе — деление, третье — сложение. Вывод: порядок действий определен неверно. Исправим ошибки, найдем значение выражения.

    18:(11-5)+47=18:6+47=3+47=50

    В этом выражении также имеются скобки, значит, сначала выполняем действие в скобках, затем слева направо умножение или деление, сложение или вычитание. Проверяем: первое действие — в скобках, второе — умножение, третье — вычитание. Вывод: порядок действий определен неверно. Исправим ошибки, найдем значение выражения.

    7*3-(16+4)=7*3-20=21-20=1

    Выполним задание.

    Расставим порядок действий в выражении, используя изученное правило (рис. 5).

    Рис. 5. Порядок действий

    Мы не видим числовых значений, поэтому не сможем найти значение выражений, однако потренируемся применять изученное правило.

    Действуем по алгоритму.

    В первом выражении имеются скобки, значит, первое действие в скобках. Затем слева направо умножение и деление, потом слева направо вычитание и сложение.

    Во втором выражении также имеются скобки, значит, первое действие выполняем в скобках. После этого слева направо умножение и деление, после этого — вычитание.

    Проверим себя (рис. 6).

    Рис. 6. Порядок действий

    Сегодня на уроке мы познакомились с правилом порядка выполнения действий в выражениях без скобок и со скобками.

    Список литературы

    1. М.И. Моро, М.А. Бантова и др. Математика: Учебник. 3 класс: в 2-х частях, часть 1. — М.: «Просвещение», 2012.
    2. М.И. Моро, М.А. Бантова и др. Математика: Учебник. 3 класс: в 2-х частях, часть 2. — М.: «Просвещение», 2012.
    3. М.И. Моро. Уроки математики: Методические рекомендации для учителя. 3 класс. — М.: Просвещение, 2012.
    4. Нормативно-правовой документ. Контроль и оценка результатов обучения. — М.: «Просвещение», 2011.
    5. «Школа России»: Программы для начальной школы. — М.: «Просвещение», 2011.
    6. С.И. Волкова. Математика: Проверочные работы. 3 класс. — М.: Просвещение, 2012.
    7. В.Н. Рудницкая. Тесты. — М.: «Экзамен», 2012.
    1. Festival.1september.ru ().
    2. Sosnovoborsk-soobchestva.ru ().
    3. Openclass.ru ().

    Домашнее задание

    1. Определи порядок действий в данных выражениях. Найди значение выражений.

    2. Определи, в каком выражении такой порядок выполнения действий:

    1. умножение; 2. деление;. 3. сложение; 4. вычитание; 5. сложение. Найди значение данного выражения.

    3. Составь три выражения, в которых такой порядок выполнения действий:

    1. умножение; 2. сложение; 3. вычитание

    1. сложение; 2. вычитание; 3. сложение

    1. умножение; 2. деление; 3. сложение

    Найди значение этих выражений.

    Октябрь 24th, 2017 admin

    Лопатко Ирина Георгиевна

    Цель: формирование знаний о порядке выполнения арифметических действий в числовых выражениях без скобок и со скобками, состоящих из 2-3 действий.

    Задачи:

    Образовательная: формировать у учащихся умение пользоваться правилами порядка выполнения действий при вычислении конкретных выражений, умение применять алгоритм действий.

    Развивающая: развивать навыки работы в паре, мыслительную деятельность учащихся, умение рассуждать, сопоставлять и сравнивать, навыки вычисления и математическую речь.

    Воспитательная: воспитывать интерес к предмету, толерантное отношение друг к другу, взаимное сотрудничество.

    Типа: изучение нового материала

    Оборудование: презентация, наглядности, раздаточный материал, карточки, учебник.

    Методы: словесный, наглядно- образный.

    ХОД УРОКА

    1. Организационный момент

    Приветствие.

    Мы сюда пришли учиться,

    Не лениться, а трудиться.

    Работаем старательно,

    Слушаем внимательно.

    Маркушевич сказал великие слова: “Кто с детских лет занимается математикой, тот развивает внимание, тренирует свой мозг, свою волю, воспитывает настойчивость и упорство в достижении цели .” Добро пожаловать на урок математики!

    1. Актуализация знаний

    Предмет математики столь серьезен, что не следует упускать ни одной возможности сделать его более занимательным. (Б. Паскаль)

    Предлагаю выполнить логические задания. Вы готовы?

    Какие два числа, если их перемножить, дают такой же результат, что и при их сложении? (2 и 2)

    Из-под забора видно 6 пар лошадиных ног. Сколько этих животных во дворе? (3)

    Петух, стоя на одной ноге весит 5кг. Сколько он будет весить, стоя на двух ногах? (5кг)

    На руках 10 пальцев. Сколько пальцев на 6 руках? (30)

    У родителей 6 сыновей. Каждый имеет сестру. Сколько всего детей в семье? (7)

    Сколько хвостов у семи котов?

    Сколько носов у двух псов?

    Сколько ушей у 5 малышей?

    Ребята, именно такой работы я и ждала от вас: вы были активны, внимательны, сообразительны.

    Оценивание: словесное.

    Устный счет

    КОРОБКА ЗНАНИЙ

    Произведение чисел 2 * 3, 4 * 2;

    Частные чисел 15: 3, 10:2;

    Сумма чисел 100 + 20, 130 + 6, 650 + 4;

    Разность чисел 180 – 10, 90 – 5, 340 – 30.

    Компоненты умножения, деления, сложения, вычитания.

    Оценивание: ученики самостоятельно оценивают друг друга

    1. Сообщение темы и цели урока

    “Чтобы переварить знания, надо поглощать их с аппетитом.” (А.Франц)

    Вы готовы поглощать знания с аппетитом?

    Ребята, Маше и Мише была предложена такая цепочка

    24 + 40: 8 – 4=

    Маша её решила так:

    24 + 40: 8 – 4= 25 правильно? Ответы детей.

    А Миша решил вот так:

    24 + 40: 8 – 4= 4 правильно? Ответы детей.

    Что вас удивило? Вроде и Маша и Миша решили правильно. Тогда почему ответы у них разные?

    Они считали в разном порядке, не договорились, в каком порядке будут считать.

    От чего зависит результат вычисления? От порядка.

    Что вы видите в этих выражениях? Числа, знаки.

    Как в математике называют знаки? Действия.

    О каком порядке не договорились ребята? О порядке действий.

    Что мы будем изучать на уроке? Какая тема урока?

    Мы будем изучать порядок арифметических действий в выражениях.

    Для чего нам нужно знать порядок действий? Правильно выполнять вычисления в длинных выражениях

    «Корзина знаний» . (Корзина висит на доске)

    Ученики называют ассоциации связанные с темой.

    1. Изучение нового материала

    Ребята, послушайте, пожалуйста, что говорил французский математик Д.Пойя: “Лучший способ изучить что-либо — это открыть самому”. Вы готовы к открытиям?

    180 – (9 + 2) =

    Прочитайте выражения. Сравните их.

    Чем похожи? 2 действия, числа одинаковые

    Чем отличаются? Скобки, разные действия

    Правило 1.

    Прочитайте правило на слайде. Дети читают вслух правило.

    В выражениях без скобок, содержащих только сложение и вычитание или умножение и деление, действия выполняются в том порядке, как они записаны: слева направо.

    О каких действиях здесь говорится? +, — или : , ·

    Из данных выражений найдите только те, которые соответствуют правилу 1. Запишите их в тетрадь.

    Вычислите значения выражений.

    Проверка.

    180 – 9 + 2 = 173

    Правило 2.

    Прочитайте правило на слайде.

    Дети читают вслух правило.

    В выражениях без скобок сначала выполняются по порядку слева направо умножение или деление, а потом сложение или вычитание.

    :, · и +, — (вместе)

    Есть скобки? Нет.

    Какие действия будем выполнять сначала? ·, : слева направо

    Какие действия будем выполнять потом? +, — слева, направо

    Найдите их значения.

    Проверка.

    180 – 9 * 2 = 162

    Правило 3

    В выражениях со скоб­ками, сна­ча­ла вы­чис­ля­ют зна­че­ние вы­ра­же­ний в скоб­ках, затем выполняются по порядку слева направо умножение или деление, а потом сложение или вычитание.

    А здесь какие арифметические действия указаны?

    :, · и +, — (вместе)

    Есть скобки? Да.

    Какие действия будем выполнять сначала? В скобках

    Какие действия будем выполнять потом? ·, : слева направо

    А затем? +, — слева, направо

    Выпишите выражения, которые относятся ко второму правилу.

    Найдите их значения.

    Проверка.

    180: (9 * 2) = 10

    180 – (9 + 2) = 169

    Еще раз все вместе проговариваем правило.

    ФИЗМИНУТКА

    1. Закрепление

    “Много из математики не остается в памяти, но когда поймешь ее, тогда легко при случае вспомнить забытое.” , говорил М.В. Остроградский. Вот и мы сейчас вспомним, что мы только что изучили и применим новые знания на практике.

    Страница 52 №2

    (52 – 48) * 4 =

    Страница 52 №6 (1)

    Учащиеся собрали в теплице 700 кг овощей: 340 кг огурцов, 150 кг помидоров, а остальные – перец. Сколько килограммов перца собрали учащиеся?

    О чем говорится? Что известно? Что нужно найти?

    Давайте попробуем решить эту задачу выражением!

    700 – (340 + 150) = 210 (кг)

    Ответ: 210 кг перца собрали учащиеся.

    Работа в парах.

    Даны карточки с заданием.

    5 + 5 + 5 5 = 35

    (5+5) : 5 5 = 10

    Оценивание:

    • быстрота – 1 б
    • правильность — 2 б
    • логичность – 2 б
    1. Домашнее задание

    Страница 52 № 6 (2) решить задачу, записать решение в виде выражения.

    1. Итог, рефлексия

    Кубик Блума

    Назови тему нашего урока?

    Объясни порядок выполнения действий в выражениях со скобками.

    Почему важно изучать эту тему?

    Продолжи первое правило.

    Придумай алгоритм выполнения действий в выражениях со скобками.

    “Если вы хотите участвовать в большой жизни, то наполняйте свою голову математикой, пока есть к тому возможность. Она окажет вам потом огромную помощь во всей вашей работе.” (М.И. Калинин)

    Спасибо за работу на уроке!!!

    ПОДЕЛИТЬСЯ Вы можете

    На данном уроке подробно рассмотрен порядок выполнения арифметических действий в выражениях без скобок и со скобками. Учащимся предоставляется возможность в ходе выполнения заданий определить, зависит ли значение выражений от порядка выполнения арифметических действий, узнать отличается ли порядок арифметических действий в выражениях без скобок и со скобками, потренироваться в применении изученного правила, найти и исправить ошибки, допущенные при определении порядка действий.

    В жизни мы постоянно выполняем какие-либо действия: гуляем, учимся, читаем, пишем, считаем, улыбаемся, ссоримся и миримся. Эти действия мы выполняем в разном порядке. Иногда их можно поменять местами, а иногда нет. Например, собираясь утром в школу, можно сначала сделать зарядку, затем заправить постель, а можно наоборот. Но нельзя сначала уйти в школу, а потом надеть одежду.

    А в математике обязательно ли выполнять арифметические действия в определенном порядке?

    Давайте проверим

    Сравним выражения:
    8-3+4 и 8-3+4

    Видим, что оба выражения совершенно одинаковы.

    Выполним действия в одном выражения слева направо, а в другом справа налево. Числами можно проставить порядок выполнения действий (рис. 1).

    Рис. 1. Порядок действий

    В первом выражении мы сначала выполним действие вычитания, а затем к результату прибавим число 4.

    Во втором выражении сначала найдем значение суммы, а потом из 8 вычтем полученный результат 7.

    Видим, что значения выражений получаются разные.

    Сделаем вывод: порядок выполнения арифметических действий менять нельзя .

    Узнаем правило выполнения арифметических действий в выражениях без скобок.

    Если в выражение без скобок входят только сложение и вычитание или только умножение и деление, то действия выполняют в том порядке, в каком они написаны.

    Потренируемся.

    Рассмотрим выражение

    В этом выражении имеются только действия сложения и вычитания. Эти действия называют действиями первой ступени .

    Выполняем действия слева направо по порядку (рис. 2).

    Рис. 2. Порядок действий

    Рассмотрим второе выражение

    В этом выражении имеются только действия умножения и деления — это действия второй ступени.

    Выполняем действия слева направо по порядку (рис. 3).

    Рис. 3. Порядок действий

    В каком порядке выполняются арифметические действия, если в выражении имеются не только действия сложения и вычитания, но и умножения и деления?

    Если в выражение без скобок входят не только действия сложения и вычитания, но и умножения и деления, или оба этих действия, то сначала выполняют по порядку (слева направо) умножение и деление, а затем сложение и вычитание.

    Рассмотрим выражение.

    Рассуждаем так. В этом выражении имеются действия сложения и вычитания, умножения и деления. Действуем по правилу. Сначала выполняем по порядку (слева направо) умножение и деление, а затем сложение и вычитание. Расставим порядок действий.

    Вычислим значение выражения.

    18:2-2*3+12:3=9-6+4=3+4=7

    В каком порядке выполняются арифметические действия, если в выражении имеются скобки?

    Если в выражении имеются скобки, то сначала вычисляют значение выражений в скобках.

    Рассмотрим выражение.

    30 + 6 * (13 — 9)

    Мы видим, что в этом выражении имеется действие в скобках, значит, это действие выполним первым, затем по порядку умножение и сложение. Расставим порядок действий.

    30 + 6 * (13 — 9)

    Вычислим значение выражения.

    30+6*(13-9)=30+6*4=30+24=54

    Как нужно рассуждать, чтобы правильно установить порядок арифметических действий в числовом выражении?

    Прежде чем приступить к вычислениям, надо рассмотреть выражение (выяснить, есть ли в нём скобки, какие действия в нём имеются) и только после этого выполнять действия в следующем порядке:

    1. действия, записанные в скобках;

    2. умножение и деление;

    3. сложение и вычитание.

    Схема поможет запомнить это несложное правило (рис. 4).

    Рис. 4. Порядок действий

    Потренируемся.

    Рассмотрим выражения, установим порядок действий и выполним вычисления.

    43 — (20 — 7) +15

    32 + 9 * (19 — 16)

    Будем действовать по правилу. В выражении 43 — (20 — 7) +15 имеются действия в скобках, а также действия сложения и вычитания. Установим порядок действий. Первым действием выполним действие в скобках, а затем по порядку слева направо вычитание и сложение.

    43 — (20 — 7) +15 =43 — 13 +15 = 30 + 15 = 45

    В выражении 32 + 9 * (19 — 16) имеются действия в скобках, а также действия умножения и сложения. По правилу первым выполним действие в скобках, затем умножение (число 9 умножаем на результат, полученный при вычитании) и сложение.

    32 + 9 * (19 — 16) =32 + 9 * 3 = 32 + 27 = 59

    В выражении 2*9-18:3 отсутствуют скобки, зато имеются действия умножения, деления и вычитания. Действуем по правилу. Сначала выполним слева направо умножение и деление, а затем от результата, полученного при умножении, вычтем результат, полученный при делении. То есть первое действие — умножение, второе — деление, третье — вычитание.

    2*9-18:3=18-6=12

    Узнаем, правильно ли определен порядок действий в следующих выражениях.

    37 + 9 — 6: 2 * 3 =

    18: (11 — 5) + 47=

    7 * 3 — (16 + 4)=

    Рассуждаем так.

    37 + 9 — 6: 2 * 3 =

    В этом выражении скобки отсутствуют, значит, сначала выполняем слева направо умножение или деление, затем сложение или вычитание. В данном выражении первое действие — деление, второе — умножение. Третье действие должно быть сложение, четвертое — вычитание. Вывод: порядок действий определен верно.

    Найдем значение данного выражения.

    37+9-6:2*3 =37+9-3*3=37+9-9=46-9=37

    Продолжаем рассуждать.

    Во втором выражении имеются скобки, значит, сначала выполняем действие в скобках, затем слева направо умножение или деление, сложение или вычитание. Проверяем: первое действие — в скобках, второе — деление, третье — сложение. Вывод: порядок действий определен неверно. Исправим ошибки, найдем значение выражения.

    18:(11-5)+47=18:6+47=3+47=50

    В этом выражении также имеются скобки, значит, сначала выполняем действие в скобках, затем слева направо умножение или деление, сложение или вычитание. Проверяем: первое действие — в скобках, второе — умножение, третье — вычитание. Вывод: порядок действий определен неверно. Исправим ошибки, найдем значение выражения.

    7*3-(16+4)=7*3-20=21-20=1

    Выполним задание.

    Расставим порядок действий в выражении, используя изученное правило (рис. 5).

    Рис. 5. Порядок действий

    Мы не видим числовых значений, поэтому не сможем найти значение выражений, однако потренируемся применять изученное правило.

    Действуем по алгоритму.

    В первом выражении имеются скобки, значит, первое действие в скобках. Затем слева направо умножение и деление, потом слева направо вычитание и сложение.

    Во втором выражении также имеются скобки, значит, первое действие выполняем в скобках. После этого слева направо умножение и деление, после этого — вычитание.

    Проверим себя (рис. 6).

    Рис. 6. Порядок действий

    Сегодня на уроке мы познакомились с правилом порядка выполнения действий в выражениях без скобок и со скобками.

    Список литературы

    1. М.И. Моро, М.А. Бантова и др. Математика: Учебник. 3 класс: в 2-х частях, часть 1. — М.: «Просвещение», 2012.
    2. М.И. Моро, М.А. Бантова и др. Математика: Учебник. 3 класс: в 2-х частях, часть 2. — М.: «Просвещение», 2012.
    3. М.И. Моро. Уроки математики: Методические рекомендации для учителя. 3 класс. — М.: Просвещение, 2012.
    4. Нормативно-правовой документ. Контроль и оценка результатов обучения. — М.: «Просвещение», 2011.
    5. «Школа России»: Программы для начальной школы. — М.: «Просвещение», 2011.
    6. С.И. Волкова. Математика: Проверочные работы. 3 класс. — М.: Просвещение, 2012.
    7. В.Н. Рудницкая. Тесты. — М.: «Экзамен», 2012.
    1. Festival.1september.ru ().
    2. Sosnovoborsk-soobchestva.ru ().
    3. Openclass.ru ().

    Домашнее задание

    1. Определи порядок действий в данных выражениях. Найди значение выражений.

    2. Определи, в каком выражении такой порядок выполнения действий:

    1. умножение; 2. деление;. 3. сложение; 4. вычитание; 5. сложение. Найди значение данного выражения.

    3. Составь три выражения, в которых такой порядок выполнения действий:

    1. умножение; 2. сложение; 3. вычитание

    1. сложение; 2. вычитание; 3. сложение

    1. умножение; 2. деление; 3. сложение

    Найди значение этих выражений.

    Правила порядка выполнения действий в сложных выражениях изучаются во 2 классе, но практически некоторые из них дети используют еще в 1 классе.

    Сначала рассматривается правило о порядке выполнения действий в выражениях без скобок, когда над числами производят либо только сложение и вычитание, либо только умножение и деление. Необходимость введения выражений, содержащих два и более арифметических действий одной ступени, возникает при знакомстве учеников с вычислительными приемами сложения и вычитания в пределах 10, а именно:

    Аналогично: 6 — 1 — 1, 6 — 2 — 1, 6 — 2 — 2.

    Так как для нахождения значений этих выражений школьники обращаются к предметным действиям, которые выполняются в определенном порядке, то они легко усваивают тот факт, что арифметические действия (сложение и вычитание), которые имеют место в выражениях, выполняются последовательно слева направо.

    С числовыми выражениями, содержащими действия сложения и вычитания, а также скобки, учащиеся впервые встречаются в теме «Сложение и вычитание в пределах 10». Когда дети встречаются с такими выражениями в 1 классе, например: 7 — 2 + 4, 9 — 3 — 1 , 4 +3 — 2; во 2 классе, например: 70 — 36 +10, 80 — 10 — 15, 32+18 — 17; 4*10:5, 60:10*3, 36:9*3, учитель показывает, как читают и записывают такие выражения и как находят их значение (например, 4*10:5 читают: 4 умножить на 10 и полученный результат разделить на 5). К моменту изучения во 2 классе темы «Порядок действий» учащиеся умеют находить значения выражений этого вида. Цель работы на данном этапе — опираясь практические умения учащихся, обратить их внимание на порядок выполнения действий в таких выражениях и сформулировать соответствующее правило. Учащиеся самостоятельно решают подобранные учителем примеры и объясняют, в каком порядке выполняли; действия в каждом примере. Затем формулируют сами или читают по учебнику вывод: если в выражении без скобок указаны только действия сложения и вычитания (или только действия умножения и деления), то их выполняют в том порядке, в каком они записаны (т.е. слева направо).

    Несмотря на то, что в выражениях вида а+в+с, а+(в+с) и (а+в)+с наличие скобок не влияет на порядок выполнения действий в силу сочетательного закона сложения, на этом этапе учащихся целесообразнее сориентировать на то, что сначала выполняется действие в скобках. Это связано с тем, что для выражений вида а — (в+с) и а — (в — с) такое обобщение неприемлемо и учащимся на начальном этапе довольно трудно будет сориентироваться в назначении скобок для различных числовых выражений. Использование скобок в числовых выражениях, содержащих действия сложения и вычитания, в дальнейшем получает свое развитие, которое связано с изучением таких правил, как прибавление суммы к числу, числа к сумме, вычитание суммы из числа и числа из суммы. Но при первом знакомстве со скобками важно нацелить учащихся на то, что сначала выполняется действие в скобках.

    Учитель обращает внимание детей на то, как важно соблюдать это правило при вычислениях, иначе можно получить неверное равенство. Например, учащиеся объясняют, каким образом, получены значения выражений: 70 — 36 +10=24, 60:10 — 3 =2, почему они неверны, какие значения в действительности имеют эти выражения. Аналогично изучают порядок действий в выражениях со скобками вида: 65 — (26 — 14), 50:(30 — 20), 90:(2 * 5). С такими выражениями учащиеся также знакомы и умеют их читать, записывать и вычислять их значение. Объяснив порядок выполнения действий в нескольких таких выражениях, дети формулируют вывод: в выражениях со скобками первым выполняется действие над числами, записанными в скобках. Рассматривая эти выражения нетрудно показать, что действия в них выполняются не в том порядке, в каком записаны; чтобы показать другой порядок их выполнения, и использованы скобки.

    Следующим вводится правило порядка выполнения действий в выражениях без скобок, когда в них содержатся действия первой и второй ступени. Поскольку правила порядка действий приняты по договоренности, учитель сообщает их детям или же учащиеся знакомятся с ними по учебнику. Чтобы учащиеся усвоили введенные правила, наряду с тренировочными упражнениями включают решение примеров с пояснением порядка выполнения их действий. Эффективны также упражнения в объяснении ошибок на порядок выполнения действий. Например, из заданных пар примеров предлагается выписать только те, где вычисления выполнены по правилам порядка действий:

    После объяснения ошибок можно дать задание: используя скобки, изменить порядок действий так, чтобы выражение имело заданное значение. Например, чтобы первое из приведенных выражений имело значение, равное 10, надо записать его так: (20+30):5=10.

    Особенно полезны упражнения на вычисление значения выражения, когда ученику приходится применять все изученные правила. Например, на доске или в тетрадях записывается выражение 36:6+3*2. Учащиеся вычисляют его значение. Затем по заданию учителя дети изменяют с помощью скобок порядок действий в выражении:

    • 36:6+3-2
    • 36:(6+3-2)
    • 36:(6+3)-2
    • (36:6+3)-2

    Интересным, но более трудным является обратное упражнение: расставить скобки так, чтобы выражение имело заданное значение:

    • 72-24:6+2=66
    • 72-24:6+2=6
    • 72-24:6+2=10
    • 72-24:6+2=69

    Также интересными являются упражнения следующего вида:

    • 1. Расставьте скобки так, чтобы равенства были верными:
    • 25-17:4=2 3*6-4=6
    • 24:8-2=4
    • 2. Поставьте вместо звездочек знаки «+» или «-» так, чтобы получились верные равенства:
    • 38*3*7=34
    • 38*3*7=28
    • 38*3*7=42
    • 38*3*7=48
    • 3. Поставьте вместо звездочек знаки арифметических действий так, чтобы равенства были верными:
    • 12*6*2=4
    • 12*6*2=70
    • 12*6*2=24
    • 12*6*2=9
    • 12*6*2=0

    Выполняя такие упражнения, учащиеся убеждаются в том, что значение выражения может измениться, если изменяется порядок действий.

    Для усвоения правил порядка действий необходимо в 3 и 4 классах включать все более усложняющиеся выражения, при вычислении значений которых ученик применял бы каждый раз не одно, а два или три правила порядка выполнения действий, например:

    • 90*8- (240+170)+190,
    • 469148-148*9+(30 100 — 26909).

    При этом числа следует подбирать так, чтобы они допускали выполнение действий в любом порядке, что создает условия для сознательного применения изученных правил.

    Почему умножение и деление «приоритетнее» сложения и вычитания

    https://a-rodionova.livejournal.com/67729.html#t592017

    Месяц назад на Рамблере появилась сообщение о том, что появившийся в твиттере пример 8:2(2+2) (https://twitter.com/pjmdolI/status/1155599063242485762 ) пользователи решают по разному: одни делят 8 на 2 и умножают на (2+2), ответ: 16, другие делят 8 на произведение 2(2+2), ответ — 1. Первых — большинство. Правильный ответ — 1. Я хотела написать пост на эту тему. Пока помещаю отрывок из него, где объясняю правила порядка действий.

    Почему умножение и деление «приоритетнее» сложения и вычитания

    Пусть выражением 20+10 =30 записано решение задачи. Слагаемыми являются известные числа, известные по условиям задачи, типа такой: вчера выкопали 20 кг моркови, сегодня — 10. Какой урожай моркови?

    Теперь представим, что слагаемые неизвестны, но по условию задачи известно, что 20 — это 10*2, а 10 — это 30:3. Так и записываем сумму: 10*2+30:3=? Нам надо получить сумму двух неизвестных чисел, чтобы подсчитать урожай.

    Для того, чтобы теперь найти сумму, нужно в первую очередь вычислить слагаемые, которыми здесь являются произведение и частное. На этом основании они становятся первоочередными действиями, а сложение — последним, заключительным действием, т. к. вычисляется искомая сумма.

    Всё очень просто. Повторю. Т.к. невозможно вычислить сумму неизвестных чисел, записанных в виде неизвестного произведения и частного без нахождения произведения и частного, то это и является той незыблемой основой первоочередного выполнения умножения и деления, когда произведение и частное являются слагаемыми. А действие сложения в таких выражениях всегда является заключительным.

    Некогда математики договорились, что для того, чтобы подобные выражения и формулы не пестрели трёхэтажными и выше скобками, не писать, а лишь подразумевать скобки для умножения и деления, т. к. произведения и частные, которые являются слагаемыми, всегда находятся в первую очередь, что всем (некогда было) ясно и без скобок (как «Волга впадает в Каспийское море»). Ставь скобки или нет, всё равно сначала будешь делить и умножать, а потом складывать и вычитать. По этим же соображениям отказались брать в скобки возведение в степень, извлечение корня и ряд других действий, которые первоочерёднее умножения и деления, когда являются неизвестными множителями, делимыми и делителями, т. е. когда неизвестно значение корня или степени, без вычисления которых нельзя совершить умножение и деление.

    Вот так надо понимать, почему умножение и деление «приоритетнее» сложения и вычитания. Первоочерёдность теперь обозначается нелепым словом «приоритетность», т. е. вы должны умножение и деление делать первыми потому, что они «приоритетнее» сложения! И самим не смешно, т. к. на деле выясняется, что умножения и деление — это вспомогательные действия, которые нужно сделать, чтобы найти сумму? Как раз сложение имеет настоящую важность. Если бы не нужно было находить сумму, то умножать/делить было бы не нужно. Будем считать, что поняли, почему умножение и деление «приоритетнее» сложения. Основу первоочерёдности умножения и деления нельзя изменить. Поэтому другие «приоритеты» будут ложными. Если вы знаете основу как смысл такой очерёдности, то плевать вы хотели и на правила и на тех, кто дурит вас при помощи своих «правилотворческих актов».

    Слово «приоритетность» теперь все понимают как обозначение некой таинственной важности умножения и деления, из-за которой им присвоили более «высокий» «приоритет». Сложение и вычитание становятся как бы «ущербными» действиями, имея самый низкий «приоритет» в «табеле о рангах». Эмоциональная нагрузка заменяет смысловую. Чтобы избавиться от эмоций, надо просто заменить слово «приоритетность» на, скажем, «очерёдность» (первоочерёдность, равноочерёдность), чтобы мозг не буксовал. Или восстановить прежнюю терминологию (не помню, что использовалось вместо «приоритетности»).

    Почему умножение и деление «равноприоритетны»

    В примере решения задачи есть действия деления и умножения. На этот случай имеется правило, что умножение и деление — равноприоритетны, т. е. их можно выполнять в произвольной последовательности.

    Действительно, абсолютно неважно, какое из слагаемых вычисляется первым, какое — вторым, т. к. очерёдность вычислений слагаемых не влияет на сумму. Неважность очерёдности действий умножения и деления при вычислении слагаемых в выражении — это основа безочерёдности этих действий, что значит, произведения и частные в выражении можно вычислять в любом порядке, как удобно, не соблюдая правило «слева направо».

    Понятно, что в выражении, где есть несколько слагаемых в виде произведений, частных, а также других действий, взятых в скобки, так же неважно для правильного ответа, в каком порядке вычислять слагаемые. Не обязательно начинать вычислять сначала все слагаемые в скобках, потом все частные и произведения, порядок — произвольный, как удобно. А также необязательно находить сразу все слагаемые, несмотря на «пониженную приоритетность» сложения/вычитания. Пожалуйста, можете выполнять сложение по мере вычисления неизвестных слагаемых. Особенно это пригождается, когда решаешь конкретную задачу, в которых промежуточные суммы имеют определённое смысловое значение, т. е. являются ответами, на промежуточные вопросы задачи. Это позволяет быстрее найти ошибку в постановке вопроса, формулировке действия или в вычислении какого-либо параметра. Если же найти все слагаемые оптом, а потом сложить, то я даже не знаю, как потом найти ошибку. Тупое исполнение правил мешает осмысленно относится к задаче и превращает решение задачи в муторный процесс вычислений (благо, его облегчили калькуляторы) и не позволяет накопить опыт (который, «сын ошибок трудных», потому трудных, что требуют исправления, но тяжело в учении — легко в бою) их решения. Правила превращают мозг человека в калькулятор.

    Почему «равноприоритетны» сложение и вычитание (на примере выражения без умножения и деления)

    Т.к. вычитание есть сложение с отрицательными числами, и от перемены мест слагаемых сумма не изменяется, то вычитание и сложение могут проводиться в любой очерёдности, т. к. это одно и то же действие. Такова основа безочерёдности выполнения действий сложения и вычитания, которую в псевдоматематическом новоязе назвали равной приоритетностью сложения и вычитания. Не может одно и то же действие быть «разноприоритетным», если уж на то пошло.

    Например: 10+20-10 можно посчитать в таком порядке: 10+(20-10), в таком: -10+10+20, в таком: -10+20+10, в таком 10-10+20. Во всех случаях ответ будет одинаков — 20. Порядок вычисления не влияет на сумму, поэтому у действий сложения и вычитания — произвольная очерёдность, которую устанавливает тот, кто находит значение выражения.

    Сложение и вычитание не имеет очерёдности согласно переместительному свойству сложения, поэтому правило «слева направо» выполнять необязательно.

    Т. к. сложение — это увеличение количества, а вычитание — уменьшение, то ребёнок не может сразу понять, почему в реальности разные действия (сложение и вычитание) формально являются одним и тем же. Поэтому самое главное, что ребёнку надо пояснить, почему разное — одинаково, так, чтобы, наоборот, не запутать его. Если он сам не поймёт, не увидит одинаковость с точки зрения именно арифметики, как бы условности исключительно для удобства счёта реальных вещей, то он просто зазубрит, мол, «что это одно и тоже», чего нельзя допускать, ибо он отчается понять, что повлечёт за собой цепь непониманий. Поэтому пояснять надо в своё время. Думаю, что одинаковость будет ясна в полной мере после понимания, что такое отрицательные числа как уяснения их назначения (практического использования) — «ниже нуля и выше нуля». До этого вычитание для ребёнка будет самостоятельным арифметическим действием, как он видит на практике, противоположностью сложения по результату. Поэтому детям сначала надо соблюдать правило «слева направо» в выражениях, где есть сложение и вычитание, чтобы получить правильный ответ. Но не делать правило догмой, а лишь подспорьем при неуверенности. Положительные слагаемые они могут складывать произвольно, они могут сложить все слагаемые, из которых потом вычитать все вычитаемые, чтобы убедиться, что сумма не меняется, и на наглядных практических примерах с теми же счётными палочками, понять — почему. В примерах, подобных 10+20-10 они могут сначала выполнить вычитание, т. е. второе действие, или сначала от 10 — 10, и всё это можно воспроизвести на практике, с помощью тех же счётных палочек. Затем в выражения можно добавлять слагаемые/вычитаемые в виде неизвестных произведений и частных. После изучения отрицательных чисел и накопления опыта действий с ними они легко могут осознать тождественность вычитания и сложения как арифметического действия, убедившись в этом на практике.

    назначение правила «слева направо»

    Мы выяснили, что если в выражении два и более слагаемых, неважно, простых или сложных, то сложение (вычитание) данных и найденных слагаемых можно производить в любом порядке. Зачем же в таких выражениях бывает нужно применять очерёдность вычисления «слева направо»?

    Иногда порядок нахождения суммы «слева направо» имеет смысл. Любое математическое, как арифметическое, так и алгебраическое, выражение определяется и составляется исходя из условий задачи. Поэтому составление и запись выражения отражает логику решения данной задачи, последовательность ответов на предварительные вопросы, получив которые, человек может получить ответ, ради которого он решал задачу и даже ставил её. Поэтому каждый член выражения, являющийся количественной характеристикой, имеет и смысловое значение, отвечая на вопрос: «количество чего?» (только в абстрактных примерах этот вопрос не ставится, число имеет только величину, или «значение», и не имеет качественной, или смысловой, характеристики). Т. к. арифметические действия записываются в порядке осознания задачи и решения предварительных вопросов, то тем самым фиксируется смысловая нагрузка членов выражения, следовательно, фиксируется смысл действия — на какой вопрос будет получен ответ. Следовательно, очередность действий в порядке записи выражения («слева направо») позволяет решающему задачу человеку сохранять логику решения, последовательно отвечая на предварительные вопросы. Только для этой цели требуется соблюдать очерёдность «слева направо». Но и в этом случае, уверенный в себе человек, хорошо понимающий смысл задачи, может не соблюдать этого правила, если ему удобно считать (ведь считать не означает — решать) в другой последовательности, как какой-либо абстрактный пример. Для этой же цели учащиеся осваивают способы «упрощения выражений» и свойства арифметических действий. Такой человек всегда может объяснить метод своего решения, как свои допущенные на время условности. Большинство людей уверенность путают с самоуверенностью, поэтому для страховки им лучше соблюдать очерёдность действий, чтобы не запутаться в задаче и не получить абсурд в виде «полтораземлекопа». В физике адекватность составленного выражения решению задачи проверяется размерностью.

    Выше я уже показала, что произвольность в очерёдности (а также хоть «справа налево») не вредит вычислению ответа, когда находятся неизвестные произведения и частные.

    В выражениях, где члены НЕ являются слагаемыми, например 8:2*4, нужно выполнять действия «слева направо». И теперь уже не только ради сохранения смысла членов выражения, а потому, что другой порядок действий даст неправильный ответ. Правило «слева направо» придаёт строгую очерёдность «безочерёдным» делению и умножению. Почему?

    Хотя умножение и деление имеют своими корнями сложение/вычитание, но в отличие от вычитания и сложения, они не являются одним и тем же действием. Умножение — это сложение одинаковых чисел, а деление — это разложение суммы на равные количественные доли. Как говорят — обратное действие. В данном примере 8 делится пополам. Одна часть = 4. Эта часть обратно складывается, но не 2 раза, чтобы опять получилась 8, а 4 раза, что в сумме даёт 16. Взаимосвязь, как обратимость, деления и умножения видна в примерах, где делитель равен множителю: 8:2*2=8. Мы разделили 8 на 2 части, потом часть сложили 2 раза, и получили 8. В общем, насчёт обратимости понятно: на сколько частей разобрали, столько и собрали. Не в этом дело. Но во взаимосвязь умножения и деления дети тоже должны вникнуть, выявить её на опыте (упражнениях), а не просто знать о ней, т. к. без этого не смогут владеть этими инструментами математики в полной мере.

    Из примера видно, что 8 является делимым, а 2 является делителем. Делитель, как теперь называется, это — «оператор действия», т.е. это то, что делит (на определённое его величиной количество частей). Поэтому 2 не может быть одновременно множителем (точнее — умножаемым) для 4. Не должно быть «или-или», т. е. двусмысленности назначения члена выражения. Множителем (умножаемым) для 4 (здесь 4 — оператор умножения) станет частное от деления 8 на 2, т. е. тоже 4. Следовательно, последовательность записи действий слева направо определяет так сказать статус каждого члена выражения: что есть делимое, делитель, множитель. А это не просто «статус», а так сказать, «положение обязывает». Делителю — делить, умножаемому — умножаться, множителю — умножать. Значит, в порядке записи обозначен порядок действий. Сама запись есть способ обозначения порядка действий. Способ, который устраняет неоднозначность, порождаемую безочерёдностью («равноприоритетностью») умножения и деления. Это есть основание правила «слева направо». Менять этот порядок, например выполнять сначала умножение 2*4, значит — фактически решать не данный, а другой пример, в котором бывший делитель (2) становится множителем для 4, а полученное произведение — делителем 8. В первом случае мы находим произведение, во-втором, частное. Т.е. изменение порядка действий изменяет пример. Другой пример — другой ответ. Если же нам нужен именно «другой пример», то нет проблем — произведение 2*4 берётся в скобки: 8:(2*4), или знак деления заменяется на горизонтальную черту. В данном случае скобки «аннулируют» очерёдность «слева направо», т. к. меняют «статус» двойки с делителя, данный ему порядком записи, на умножаемое. Чтобы оно благополучно умножилось. В алгебраических выражениях, типа a:bc, чтобы обозначить делителем b, нужно делимое и делитель взять в скобки(а:b)c, или заменить знак деления на горизонтальный. Хотя проще всего в таких случаях поставить знак умножения между b*c. Точнее, при записи алгебраического выражения его просто не надо опускать. Но принято его опускать, поэтому скобки — в помощь.

    ***

    Я разъяснила объективные основания «приоритетности» всех арифметических действий. Надеюсь теперь всем понятно, почему не может быть ни различных, ни других правил «приоритетности». Порядок действий не зависит от человека. От человека зависит лишь его формулировка в виде правил. При желании каждый может формулировать правила «своими словами», формулируя своё понимание очерёдности.

    Повторю, что это отрывок, где я показываю смысл правил BODMAS/PEMDAS, т.к. без смысла они становятся догмой. Но весь сыр-бор разгорелся по причине того, что никто, включая решивших правильно, не понимает смысла опущенного перед скобками знака умножения. Опущенный знак умножения имеет назначение скобок, поэтому в «спорном примере» делителем 8 является произведение 2(2+2). Или, выражаясь на математическом новоязе, опущенный знак умножения делает действие умножения «приоритетнее» деления, т.к. согласно правилам, действия в скобках первичны. В данном случае опущенный знак умножения меняет порядок вычисления «слева направо» на «справа налево», определяя делителем произведение. Постараюсь дописать текст, т.к. мне надо было понять, как вообще и почему мог произойти такой «спор». Для меня это как гром с ясного неба.


    Телеграмм-канал для своих, не скопипащенных, постов: t.me/warrax_news

    Зубодробительная задачка с очень простой математикой

    В интернете много споров про такие примеры, поэтому мы решили разобраться, какие ошибки совершают чаще всего и почему многие считают неправильно. Для решения нам понадобятся три математических правила:

    1. То, что в скобках, выполняется в первую очередь. Если скобок несколько, они выполняются слева направо.
    2. При отсутствии скобок математические действия выполняются слева направо, сначала умножение и деление, потом — сложение и вычитание.
    3. Между множителем и скобкой (или двумя скобками) может опускаться знак умножения.

    Разберём подробнее, что это значит в нашем случае.

    1. То, что в скобках, выполняется в первую очередь. То есть в нашем примере, вне зависимости от чего угодно, сначала схлопнутся скобки:

    8 / 2(2 + 2) → 8 / 2(4)

    2. Между числом и скобкой можно опустить знак умножения. У нас перед скобкой двойка, то есть можно сделать такую замену:

    8 / 2(4) → 8 / 2 × 4

    3. Математические действия при отсутствии скобок выполняются слева направо: как при чтении, сначала умножение и деление, потом — сложение и вычитание. Умножение и деление имеют одинаковый приоритет. Нет такого, что сначала всегда делается умножение, затем деление, или наоборот. Со сложением и вычитанием то же самое.

    Некоторые считают, что раз множители были написаны близко друг к другу (когда там стояли скобки), то оно выполняется в первую очередь, ссылаясь при этом на разные методические пособия. На самом деле это не так, и нет такого скрытого умножения, которое имеет приоритет над другим умножением или делением. Это такое же умножение, как и остальные, и оно делается в общем порядке — как и принято во всём математическом мире.

    Получается, что нам сначала надо сложить 2 + 2 в скобках, потом 8 разделить на 2, и полученный результат умножить на то, что в скобках:

    8 / 2 × (2 + 2) = 8 / 2 × 4 = 4 × 4 = 16

    Кстати, если на айфоне записать это выражение точно так же, как в условии, телефон тоже даст правильный ответ.

    А инженерный калькулятор на Windows 10 так записывать не умеет и пропускает первую двойку-множитель. Попробуйте сами 🙂

    Тут в тред врываются математики и с воплями «Шустеф!» поясняют криком:

    «В АЛГЕБРЕ ТОТ ЖЕ ПОРЯДОК ДЕЙСТВИЙ, ЧТО И В АРИФМЕТИКЕ, но есть исключение: в алгебре знак умножения связывает компоненты действия сильнее, чем знак деления, поэтому знак умножения опускается. Например, a:b·c= a: (b·c)».

    Этот текст из «Методики преподавания алгебры», курс лекций, Шустеф М. Ф., 1967 год. (стр. 43)

    Раз в спорном примере знак умножения опущен, то спорный пример алгебраический, а значит, сначала умножаем 2 на 4, а потом 8 делим на 8!

    Та самая цитата.

    А вот как на это отвечают те, кто действительно в теме и не ленится полностью посмотреть первоисточник:

    «Для устранения недоразумений В. Л. Гончаров указывает, что предпочтительнее пользоваться в качестве знака деления чертой и ставить скобки [87]. П. С. Александров и А. Н. Колмогоров [59] предложили изменить порядок действий в арифметике и решать, например, так: 80:20×2=80:40=2 вместо обычного: 80:20×2=4×2=8. Однако это предложение не нашло поддержки».

    Если апеллировать к Фриде Максовне Шустеф, то выходит, что:

    1. В. Л. Гончаров говорит так: «Ребята, используйте черту и ставьте скобки, чтобы ни у кого не было вопросов про приоритет».
    2. Если у нас всё же битва арифметики и алгебры, то, по П. С. Александрову и А. Н. Колмогорову, пример нужно решать слева направо, как обычно. Они, конечно, предложили решать такое по-другому, но научное сообщество их не поддержало.

    Самое интересное, что дальше в примерах Фрида Максовна пользуется как раз правильным порядком действий, объясняя решение. Даже там, где есть умножение на скобку с опущенным знаком, она выполняет действия слева направо.

    Полная цитата из Шустеф, которая, оказывается, имеет в виду совсем не то.

    Порядок операций — PEDMAS

    Порядок операций можно определить как стандартную процедуру, которая указывает, какие вычисления следует начинать в выражении с несколькими арифметическими операциями. Без согласованного порядка работы можно совершить большие ошибки во время вычислений.

    Например, выражение, которое влечет за собой больше, чем операцию, такую ​​как вычитание, сложение, умножение или деление, требует стандартного метода определения того, какую операцию выполнить первой.

    Например, если вы хотите решить такую ​​проблему, как; 5 + 2 x 3, возникает проблема, какая операция запускается первой?

    Поскольку у этой проблемы есть два варианта решения, какой ответ правильный?

    Если мы сначала выполняем сложение, а затем умножение, результат будет:

    5 + 2 x 3 = (5 + 2) x 3 = 10 x 3 = 30

    Если мы сначала выполняем умножение, а затем сложение, результат:

    5 + 2 x 3 = 5 + (2 x 3) = 5 + 6 = 11

    Чтобы узнать, какой из них является правильным, есть мнемоническое слово «PEMDAS», которое полезно, поскольку оно напоминает нам правильного порядка действий.

    PEMDAS

    PEMDAS — это аббревиатура, обозначающая скобки, экспоненты, умножение, сложение и вычитание. Порядок действий:

    • P для круглых скобок: (), скобок [], фигурных скобок {} и дробных черт.
    • E — экспонента, включая корни.
    • M для отдела.
    • D для умножения.
    • A — для дополнения.
    • S — вычитание.

    Правила PEMDAS

    • Всегда начинайте с вычисления всех выражений в скобках
    • Упростите все экспоненты, такие как квадратные корни, квадраты, кубы и корни куба
    • Выполните умножение и деление, начиная слева направо
    • Наконец, проделайте сложение и вычитание аналогично, начиная слева направо.

    Один из способов освоить этот порядок работы — вспомнить любую из следующих трех фраз; Выберите тот, который вам легче запомнить.

    • «P аренда E xcuse M y D ухо A Unt S »
    • «Большие слоны уничтожают мышей и улиток».
    • «Розовые слоны уничтожают мышей и улиток».

    Пример 1

    Решить

    30 ÷ 5 x 2 + 1

    Решение

    Поскольку скобок и экспонент нет, начните с умножения, а затем деления слева направо.Завершите операцию сложением.

    30 ÷ 5 = 6

    6 x 2 = 12

    12 + 1 = 13

    ПРИМЕЧАНИЕ: Следует отметить, что, хотя умножение в PEMDAS предшествует делению, однако операция двух всегда слева направо.

    Выполнение умножения перед делением приводит к неправильному ответу:

    5 x 2 = 10

    30 ÷ 10 = 3

    3 + 1 = 4

    Пример 2

    Решите следующее выражение: 5 + (4 — 2) 2 x 3 ÷ 6 — 1

    Решение

    • Начните со скобок;

    (4-2) = 2

    • Перейти к экспоненциальной операции.

    2 2 = 4

    • Теперь у нас осталось; 5 + 4 x 3 ÷ 6 — 1 =?
    • Выполните умножение и деление, начиная слева направо.

    4 x 3 = 12

    5 + 12 ÷ 6 — 1

    Начиная справа;

    12 ÷ 6 = 2

    5 + 2 — 1 =?

    5 + 2 = 7

    7 — 1 =?

    7 — 1 = 6

    Пример 3

    Упростить 3 2 + [6 (11 + 1 — 4)] ÷ 8 x 2

    Решение

    Для решения этой проблемы используется PEMDAS. применяется следующим образом;

    • Начните операцию с скобок.
    • Начните внутри скобок, пока все группировки не будут устранены. Добавление сделано;

    11 + 1 = 12

    • Выполните вычитание; 12 — 4 = 8
    • Проработать кронштейны как; 6 x 8 = 48
    • Выполните экспоненты как; 3 2 = 9

    9 + 48 ÷ 8 x 2 =?

    • Выполните умножение и деление слева направо;

    48 ÷ 8 = 6

    6 x 2 = 12

    Пример 4

    Вычислить выражение; 10 ÷ 2 + 12 ÷ 2 × 3

    Решение

    При применении правила PEMDAS умножение и деление оцениваются слева направо.Желательно вставить круглые скобки, чтобы напомнить себе о порядке работы

    10 ÷ 2 + 12 ÷ 2 × 3

    = (10 ÷ 2) + (12 ÷ 2 × 3)

    = 23

    Пример 5

    Вычислить 20 — [3 x (2 + 4)]

    Решение

    Сначала определите выражения в скобках.

    = 20 — [3 x 6]

    Найдите оставшиеся скобки.
    = 20 — 18

    Наконец, выполните вычитание, чтобы получить 2 в качестве ответа.

    Пример 6

    Тренировка (6 — 3) 2 — 2 x 4

    Решение

    • Начните с раскрытия скобок

    = (3) 2 — 2 x 4

    = 9 — 2 x 4

    • Теперь произведите умножение

    = 9-8

    • Завершите операцию вычитанием, чтобы получить 1 как правильный ответ.

    Пример 7

    Решите уравнение 2 2 — 3 × (10 — 6)

    Решение

    • Вычислить в скобках.
      = 2 2 — 3 × 4
    • Определите показатель степени.
      = 4 — 3 x 4
    • Произвести умножение.
      = 4 — 12
    • Завершите операцию вычитанием.
      = -8

    Пример 8

    Упростите выражение 9-5 ÷ (8-3) x 2 + 6, используя порядок операций.

    Решение

    • Тренировка в скобках

    = 9-5 ÷ 5 x 2 + 6

    = 9 — 1 x 2 + 6

    • Произвести умножение

    = 9 — 2 + 3

    • Сложение, а затем вычитание

    = 7 + 6 = 13

    Заключение

    В заключение, иногда выражение может содержать две операции на одном уровне.

    Например, если выражение содержит и квадрат, и куб, сначала можно обработать любой из них. Всегда выполняйте операцию слева направо, следуя правилу PEMDAS. Если вы встретите выражение без символов группировки, таких как фигурные скобки, скобки и круглые скобки, вы можете упростить операцию, добавив свои собственные символы группировки.

    Работа с выражениями, содержащими дроби, решается путем упрощения сначала числителя, а затем знаменателя. Следующий шаг — по возможности упростить числитель и знаменатель.

    Практические вопросы

    1) Упростите выражение;

    2 + 3 2 (5-1)

    2) Решите

    4-3 [4-2 (6-3)] ÷ 2

    3) Упростите следующее выражение с помощью PEMDAS:

    16 — 3 (8 — 3) 2 ÷ 5

    4) Используя PEMDAS, упростите следующее алгебраическое выражение:

    14 z + 5 [6 — (2 z + 3)]

    5) Упростите алгебраическое выражение ниже;

    — {2 y — [3 — (4 — 3 y)] + 6 y

    6) Вычислите следующее выражение, используя порядок операций:

    3 + 6 x (4 + 5) ÷ 3 — 7

    7) Оцените приведенное ниже выражение с помощью PEMDAS.

    150 ÷ (6 + 3 x 8) — 5

    8) Упростите следующее выражение;

    45 ÷ (8 {5 — 4} — 3)

    Предыдущий урок | Главная страница | Следующий урок

    Термины для уравнений сложения, вычитания, умножения и деления — математика для 3-го класса

    Выучите термины для уравнений сложения, вычитания, умножения и деления

    Итак, вы научились решать уравнения сложения, вычитания, умножения и деления.👏

    Давайте рассмотрим терминов для каждого из них.

    Совет: Термины — это имен различных частей уравнения.

    Условия дополнения

    Слагаемые — это числа, которые складываются вместе.

    Сумма — это ответ, который вы получите, сложив числа.

    Мы пишем знак плюс ( +) между двумя слагаемыми и знак равенства перед суммой.

    Совет: Знак равенства (=) означает, что элементы слева и справа от него равны.

    Условия вычитания

    Minuend — это число, из которого вычитается. Это большее число.

    Subtrahend — это число, которое вычитается из уменьшаемого. Это меньшее число.

    Вычитаемое значение всегда предшествует вычитаемому .

    Совет для запоминания:

    Разница — это ответ, который мы получаем в уравнении вычитания.

    Мы используем знак минус (-) между уменьшаемым и вычитаемым.

    Запишем знак равенства перед разницей.

    Условия умножения

    Множаемое — это число, которое нужно умножить.

    Множитель — это число, указывающее, сколько раз следует умножить множимое.

    Множаемое и множитель также называются коэффициентами .

    Множитель часто записывается первым, но положение этих чисел не имеет особого значения.Это называется коммутативным свойством умножения.

    Ответ в уравнении умножения называется произведением .

    Знак умножения ( ×) записывается между двумя множителями. Его также называют знаком раз.

    Условия для подкласса

    Дивиденды — это делимое число.

    Делитель — это число, указывающее, сколько раз следует разделить дивиденд.Он отвечает на вопрос «На сколько равных групп делится число?».

    Ответ, который мы получаем в уравнении деления, называется частным .

    Знак деления (÷) помещается между делимым и делителем. Это короткая горизонтальная линия с точками над и под ней.

    Совет: Вы также можете увидеть /, используемые как знак деления. То же, что и ÷.

    Смотри и учись

    Отличная работа по изучению этих терминов.👏

    А теперь попробуйте практиковаться, чтобы убедиться, что вы помните, что они означают.

    Как происходит порядок операций в Python?

    PEMDAS P , E , MD , AS ; умножение и деление имеют одинаковый приоритет , то же самое касается сложения и вычитания. Когда оператор деления появляется перед умножением, сначала идет деление.

    Порядок выполнения операторов Python определяется приоритетом операторов и подчиняется тем же правилам.Операторы с более высоким приоритетом выполняются перед операторами с более низким приоритетом, но операторы имеют , соответствующий приоритету , когда они находятся в одной группе.

    Для 10-7 // 2 * 3 + 1 у вас есть 2 класса операторов, от низшего к высшему:

    • +, - (коррелирует с AS == сложение и вычитание )
    • *, @, /, //,% (коррелирует с MD , поэтому умножение и деление ).

    Итак, сначала выполняются // и * ; умножение и деление попадают в одну и ту же группу , здесь нет установленного порядка ( MD не означает, что умножение предшествует делению):

      10 - ((7 // 2) * 3) + 1
      

    Итак, сначала выполняется 7 // 2 , затем следует умножение на 3. Затем вы получаете вычитание из десяти и добавление единицы в конце.


    Мы упустили проблему, которая не влияет на ваш конкретный случай, но очень важна для написания реальных программ Python.PEMDAS — это не порядка операций; он не решает, в каком порядке будут оцениваться объекты. На самом деле это групп аргументов . PEMDAS говорит, что a + b + c * d оценивается как (a + b) + (c * d) , но не говорит, что сначала оценивается a + b или c * d .

    В математике не имеет значения, что вы оцениваете в первую очередь, если вы уважаете группировку аргументов. В Python, если вы сначала вычислили b () и c () в a () + (b () + c ()) только потому, что они заключены в круглые скобки, вы могли бы получить совершенно другой результат, потому что функции Python могут иметь побочные эффекты.

    Оценка выражений Python в основном выполняется слева направо. Например, в a () + b () + (c () * d ()) порядок оценки выглядит следующим образом:

    • а ()
    • б ()
    • первый + , теперь, когда его аргументы готовы
    • c ()
    • г ()
    • * , теперь, когда его аргументы готовы
    • второй + , теперь, когда его аргументы готовы

    Это несмотря на высокий приоритет * и круглые скобки вокруг умножения.

    Правило PEMDAS: понимание порядка операций

    Каждый, кто посещал математические курсы в США, раньше слышал аббревиатуру «PEMDAS». Но что именно это означает? Здесь мы подробно объясним значение PEMDAS и то, как он используется , прежде чем дать вам несколько примеров задач PEMDAS, чтобы вы могли практиковать то, что вы узнали.

    PEMDAS Значение: что это означает?

    PEMDAS — это аббревиатура, призванная помочь вам запомнить порядок операций, используемых для решения математических задач. Обычно произносится как «пем-дасс», «пем-дозз» или «пем-досс».

    Вот что означает каждая буква в PEMDAS:

    • P аренцев
    • E xponents
    • M ultiplication и D ivision
    • A ddition и S ubtraction

    Порядок букв показывает порядок, в котором вы должны решать различные части математической задачи , при этом сначала идут выражения в скобках, а последними — сложение и вычитание.

    Многие ученики используют этот мнемонический прием, чтобы помочь им запомнить каждую букву: Пожалуйста, извините, моя дорогая тетя Салли .

    В Великобритании и других странах студентов обычно изучают PEMDAS как BODMAS . Значение BODMAS такое же, как значение PEMDAS — просто используется пара разных слов. В этом аббревиатуре B обозначает «скобки» (то, что мы в США называем круглыми скобками), а O обозначает «порядки» (или показатели).

    Итак, как именно вы используете правило PEMDAS? Давайте взглянем.

    Как вы используете PEMDAS?

    PEMDAS — это аббревиатура, используемая для напоминания людям о порядке операций.

    Это означает, что вы не просто решаете математические задачи слева направо; скорее, вы решаете их в заранее определенном порядке, который предоставляется вам через аббревиатуру PEMDAS . Другими словами, вы начнете с упрощения любых выражений в круглых скобках, прежде чем упрощать любые экспоненты и переходить к умножению и т. Д.

    Но это еще не все.Вот что означает PEMDAS для решения математических задач:

    • Круглые скобки: Все, что указано в скобках, необходимо сначала упростить
    • Показатели степени: Все, что имеет показатель степени (или квадратный корень), должно быть упрощено после Все в скобках было упрощено
    • Умножение и деление: После того, как разобрались со скобками и показателями степени, решите любое умножение и деление слева направо
    • Сложение и вычитание: После того, как разобрались со скобками, показателями, умножением и делением, решите любое сложение и вычитание слева направо

    Если какой-либо из этих элементов отсутствует (например,g., у вас есть математическая задача без экспонентов), вы можете просто пропустить этот шаг и перейти к следующему.

    Теперь давайте рассмотрим пример задачи, чтобы помочь вам лучше понять правило PEMDAS:

    4 (5 — 3) ² — 10 ÷ 5 + 8

    У вас может возникнуть соблазн решить эту математическую задачу слева направо, но это приведет к неправильному ответу! Итак, вместо этого давайте использовать PEMDAS, чтобы помочь нам приблизиться к нему правильным способом .

    Мы знаем, что сначала нужно разобраться со скобками.В этой задаче заключены одни скобки: (5 — 3). Упрощение дает нам 2 , поэтому теперь наше уравнение выглядит так:

    4 (2) ² — 10 ÷ 5 + 8

    Следующая часть PEMDAS — экспоненты (и квадратные корни). В этой задаче есть один показатель степени, который возводит в квадрат число 2 (то есть то, что мы нашли, упростив выражение в скобках).

    Это дает нам 2 × 2 = 4. Итак, теперь наше уравнение выглядит так:

    4 (4) — 10 ÷ 5 + 8 OR 4 × 4 — 10 ÷ 5 + 8

    Далее идет умножение и деление слева направо .Наша задача содержит как умножение, так и деление, которые мы будем решать слева направо (сначала 4 × 4, а затем 10 ÷ 5). Это упрощает наше уравнение следующим образом:

    16-2 + 8

    Наконец, все, что нам нужно сделать, это решить оставшееся сложение и вычитание слева направо :

    16-2 + 8
    14 + 8
    = 22

    Окончательный ответ: 22. Не верите? Вставьте все уравнение в свой калькулятор (написанное в точности так, как указано выше), и вы получите тот же результат!

    Дэвид Геринг / Flickr

    Примеры математических задач с использованием PEMDAS + ответы

    Посмотрите, сможете ли вы правильно решить следующие четыре проблемы, используя правило PEMDAS.Мы рассмотрим ответы позже.

    Пример проблем PEMDAS

    1. 11-8 + 5 × 6
    2. 8 ÷ 2 (2 + 2)
    3. 7 × 4 — 10 (5 — 3) ÷ 2²
    4. √25 (4 + 2) ² — 18 ÷ 3 (3 — 1) + 2³

    ответов

    1. 33
    2. 16
    3. 23
    4. 176

    Ответ объяснения

    Здесь мы рассмотрим каждую проблему, указанную выше, и то, как вы можете использовать PEMDAS, чтобы получить правильный ответ.

    # 1 Ответ Объяснение

    11-8 + 5 × 6

    Эта математическая задача представляет собой довольно простой пример PEMDAS, который использует сложение, вычитание и умножение только , поэтому здесь не нужно беспокоиться о скобках или показателях.

    Мы знаем, что умножение предшествует сложению и вычитанию , поэтому вам нужно начать с умножения 5 на 6, чтобы получить 30:

    .

    11–8 + 30

    Теперь мы можем просто работать слева направо над сложением и вычитанием:

    11-8 + 30
    3 + 30
    = 33

    Это приводит нас к , правильный ответ — 33 .

    # 2 Ответ Объяснение

    8 ÷ 2 (2 + 2)

    Если эта математическая задача кажется вам знакомой, возможно, это связано с тем, что стал вирусным в августе 2019 года из-за неоднозначной настройки . Многие люди спорили о том, был ли правильный ответ 1 или 16, но, как мы все знаем, в математике есть (почти всегда!) Только один действительно правильный ответ .

    Так что это: 1 или 16?

    Давайте посмотрим, как PEMDAS может дать нам правильный ответ.В этой задаче есть скобки, деление и умножение. Итак, мы начнем с упрощения выражения в скобках, согласно PEMDAS:

    .

    8 ÷ 2 (4)

    В то время как большинство людей в сети до этого момента соглашались, многие не соглашались с тем, что делать дальше: умножить ли 2 на 4 или разделить 8 на 2?

    PEMDAS может ответить на этот вопрос: когда дело доходит до умножения и деления, вы всегда работаете слева направо. Это означает, что вы действительно должны разделить 8 на 2, прежде чем умножить на 4.

    Было бы полезно взглянуть на проблему таким образом, поскольку люди склонны запутаться в круглых скобках (помните, что все, что находится рядом с круглыми скобками, умножается на на то, что указано в скобках):

    8 ÷ 2 × 4

    Теперь решим уравнение слева направо:

    8 ÷ 2 × 4
    4 × 4
    = 16

    Правильный ответ — 16. Любой, кто утверждает, что это 1, определенно неправ — и явно неправильно использует PEMDAS!

    Если бы только эти примеры проблем PEMDAS были такими простыми…

    # 3 Ответ Объяснение

    7 × 4 — 10 (5 — 3) ÷ 2²

    Теперь все становится немного сложнее.

    В этой математической задаче есть скобки, показатель степени, умножение, деление, вычитание и . Но не расстраивайтесь — давайте поработаем над уравнением, шаг за шагом.

    Во-первых, согласно правилу PEMDAS, мы должны упростить то, что указано в скобках :

    7 × 4 — 10 (2) ÷ 2²

    Легко и просто, правда? Затем давайте упростим показатель степени :

    7 × 4 — 10 (2) ÷ 4

    Теперь осталось только умножение, деление и вычитание.Помните, что с умножением и делением мы просто работаем слева направо:

    7 × 4-10 (2) ÷ 4
    28-10 (2) ÷ 4
    28-20 ÷ 4
    28-5

    После того, как вы умножили и разделили, вам просто нужно сделать вычитание , чтобы решить:

    28–5
    = 23

    Это дает нам правильный ответ 23 .

    # 4 Ответ Объяснение

    √25 (4 + 2) ² — 18 ÷ 3 (3 — 1) + 2³

    Эта проблема может показаться пугающей, но я обещаю, что это не так! Если вы подходите к ней по шагам, используя правило PEMDAS , вы сможете решить ее в кратчайшие сроки.

    Сразу видно, что эта задача содержит всех компонентов PEMDAS : круглые скобки (два набора), показатели степени (два и квадратный корень), умножение, деление, сложение и вычитание. Но на самом деле это не отличается от любой другой математической задачи, которую мы решали.

    Во-первых, мы должны упростить то, что заключено в два набора круглых скобок:

    √25 (6) ² — 18 ÷ 3 (2) + 2³

    Затем мы должны упростить все экспоненты — , включая квадратные корни :

    5 (36) — 18 ÷ 3 (2) + 8

    Теперь мы должны выполнить умножение и деление слева направо:

    5 (36) — 18 ÷ 3 (2) + 8
    180 — 18 ÷ 3 (2) + 8
    180 — 6 (2) + 8
    180 — 12 + 8

    Наконец, решаем оставшееся сложение и вычитание слева направо:

    180 — 12 + 8
    168 + 8
    = 176

    Это приводит нас к и правильному ответу 176 .

    Что дальше?

    Еще одна математическая аббревиатура, которую вам следует знать — SOHCAHTOA. В нашем экспертном руководстве рассказывается, что означает аббревиатура SOHCAHTOAH и как вы можете использовать ее для решения задач, связанных с треугольниками.

    Готовитесь к разделу SAT или ACT Math? Тогда вы обязательно захотите ознакомиться с нашим полным руководством по SAT Math / ACT Math, которое дает вам множество советов и стратегий для этого сложного раздела.

    Заинтересованы в действительно больших цифрах? Узнайте, что такое гугол и гуголплекс, а также почему невозможно выписать одно из этих чисел.

    Как решать комбинированные операции

    В этом посте мы узнаем, в каком порядке нужно выполнять комбинированные операции: сложение , вычитание, умножение и деление . Комбинированные операции нельзя выполнять произвольно, необходимо соблюдать порядок.

    • Шаг 1. Выполните операции, указанные в скобках.

    Например: 3 x (2 + 4).

    Сначала мы выполняем операцию внутри скобок: 2 + 4 = 6.

    Далее выполняем операцию: 3 x 6 = 18.

    • Шаг 2: выполняйте умножение и деление всегда слева направо.

    Например: 24 ÷ 6 x 2.

    Сначала мы выполняем деление, потому что оно левее умножения: 24 ÷ 6 = 4.

    Затем мы производим умножение: 4 x 2 = 8

    • Шаг 3: Наконец, выполните сложение и вычитание.

    Например: 2 + 3 x 5.

    Сначала производим умножение: 3 x 5 = 15.

    Затем берем сумму: 2 + 15 = 17.

    Пример №1 комбинированных операций:

    6 + (8-3) х 2


    Сначала завершаем операцию в скобках: 8 — 3 = 5.

    Отсюда имеем: 6 + 5 x 2.

    Теперь произведем умножение: 5 x 2 = 10.
    И, наконец, сложение: 6 + 10 = 16.

    Пример № 2 комбинированных операций:

    21 ÷ 3 + 7 x 4

    Первая операция, которая должна быть завершена, указана в скобках, но в данном случае их нет.
    Следующее, что нужно сделать, это умножение и деление: 21/3 = 7, а на другом 7 x 4 = 28
    Теперь у нас осталось только сложение: 7 + 28 = 35

    Щелкните здесь, если хотите узнать о приемах решения комбинированных операций! Или войдите в Smartick и продолжайте изучать математику.

    Подробнее:

    Команда по созданию контента.
    Многопрофильная и многонациональная команда, состоящая из математиков, учителей, профессоров и других специалистов в области образования!
    Они стремятся создать максимально возможное математическое содержание.

    Простая математическая задача разделила Интернет | Статья

    .

    Математика: это огонь на этой неделе

    Математика может быть интересной, но может быть и сложной.

    Сложение, вычитание, умножение и деление — числа иногда действительно поражают воображение.

    И действительно, это верно для любого возраста, и на этой неделе в Интернете появилось множество мнений об ответе на эту математическую задачу:

    Пользователь Twitter с именем @pjmdoll опубликовал уравнение 8 ÷ 2 (2 + 2) =?

    Ответ… не сразу понятно, учитывая разнообразие ответов.

    Многие в Твиттере были абсолютно уверены, что ответ один.

    И многие были уверены, что ответ — 16.

    Другие были уверены, что и то, и другое.

    Но в целом многим напомнили, почему они не любят математику.

    Но каков истинный ответ, особенно если два разных калькулятора дают два разных результата?

    На самом деле все сводится к вашему подходу к математике, когда вам требуется выполнить порядок операций.

    В Канаде преподают BEDMAS, что означает, что сначала обрабатывается все, что указано в скобках, затем следуют экспоненты, деление или умножение (слева направо) и, наконец, сложение или вычитание (опять же, слева направо).

    Это означало бы, что вы должны сначала добавить в скобки (2 + 2), чтобы получить результат четыре.

    Остается 8 ÷ 2 (4) =?

    Работая слева направо, вы должны сначала позаботиться о делении, поскольку 2 (4) — это операция умножения, а не какая-то случайная работа со скобками.

    Разделив восемь на два, вы получите четыре, а это значит, что теперь ваше уравнение будет выглядеть так: 4 (4) =?

    Теперь все, что вам нужно сделать, это умножить два числа, получив в итоге 16.

    Так почему люди получают результат? Что ж, похоже, это как-то связано с порядком операций, называемым PEMDAS.

    Этот метод просит людей сначала сделать круглые скобки (скобки), затем показатели, умножение или деление, а затем сложение или вычитание.

    Но здесь все идет немного в сторону: PEMDAS по-прежнему требует от вас деления / умножения и сложения / вычитания слева направо.

    Итак, результат должен быть таким же: 16.

    Но если вы сначала умножите, как это делают некоторые, ответ будет единым.


    Math — это сложно!

    What’s Fire This Week — это ваш еженедельный дайджест того, о чем все говорят.

    Базовая математика в Excel с использованием функции СУММ

    Вы новичок в Excel и не знаете, с чего начать с формул и операций? Многие электронные таблицы используют простую математику: сложение и вычитание чисел или умножение и деление.Понимание этих математических формул поможет вам перейти к более сложным уравнениям для управления данными Excel.

    Как базовая математика работает в Excel?

    Excel использует формулы или функции для управления данными. Функция принимает входные данные, представляющие собой содержимое ячеек плюс некоторые рабочие символы, которые говорят, что делать с данными. Вы можете ввести одну ячейку в функцию или диапазон из нескольких ячеек.

    Для сложения, вычитания, умножения и деления можно использовать функцию СУММ .Ваш ввод для формулы СУММ — это базовое математическое выражение. Вот пример:

    = СУММ (A1-A2)

    В переводе на английский язык эта функция означает: «Вычислить сумму числа в ячейке A1 за вычетом числа в ячейке A2».

    Базовая математика на самом деле не требует, чтобы функция СУММ работала. В некоторых случаях вы можете пропустить формат «= СУММ ()» и просто ввести ввод функции, например «= A1 + A2». Однако использование функции СУММ иногда необходимо и рекомендуется, если вы новичок в формулах.

    Сложение чисел

    Чтобы складывать числа с помощью СУММ, используйте знак плюса (+).

    = СУММ (A1 + A2)

    Помните, что вы не ограничены простым сложением двух чисел. Вы можете сложить много чисел и связать их вместе со знаками плюс, а также вы можете сложить весь диапазон соседних ячеек с помощью двоеточия (:). Например:

    = СУММ (E1: E4)

    Эта формула означает «Вычислить сумму ячеек с E1 по E4.”

    Вычитание чисел

    Вычитание работает так же, как сложение, за исключением того, что вы используете символ минус (-).

    = СУММ (A1-A2)

    И вы можете смешать вычитание со сложением, например:

    = СУММ (A1-A2 + A3)

    Умножение чисел

    Для умножения вы используете звездочку (*) в качестве оператора умножения. В приведенном ниже примере мы умножили общее количество часов на почасовую ставку, чтобы получить окончательную сумму.Обратите внимание, что «Общее количество часов» в ячейке E5 само по себе является суммой ячеек с E1 по E4.

    = СУММ (E5 * E6)

    Разделительные числа

    Еще раз, деление работает так же, как и умножение, за исключением того, что вы используете косую черту (/) в качестве оператора деления.

    = СУММ (E7 / 2)

    Здесь мы делим сумму из ячейки E7 на два.

    Бонусный совет: сложные уравнения

    Если вы хотите связать сложение и вычитание с умножением и делением в одном уравнении, вам нужно будет использовать круглые скобки, чтобы обозначить, какие операции выполняются в первую очередь.Операции в круглых скобках выполняются первыми — и если вы используете несколько вложенных скобок, самые глубокие скобки выполняются первыми, а расчет выполняется «вовне».

    Например, вы хотите умножить сумму ячеек с E1 по E4 на сумму ячеек с F1 по F3. Вы хотите, чтобы две умножаемые суммы сначала складывались, а затем умножались.

    = СУММ ((E1: E4) * (F1: F3))

    Здесь мы используем диапазоны ячеек (например, «E1: E4») как сокращение для сложения.Функция СУММ автоматически суммирует этот диапазон ячеек. Вот пример использования вычитания перед умножением:

    = СУММ ((A1-A2) * (F1: F3))

    Итак, здесь мы вычитаем A2 из A3, а затем умножаем этот результат на сумму ячеек с F1 по F3.

    admin

    Добавить комментарий

    Ваш адрес email не будет опубликован. Обязательные поля помечены *